MathFaq.txt - Sci.Math FAQ

		   FREQUENTLY ASKED QUESTIONS IN MATHEMATICS



   The Sci.Math FAQ Team.
   Editor: Alex Lspez-Ortiz


   e-mail: alopez-o@neumann.UWaterloo.ca



   There are several versions available of this document.

   Users with graphic browsers (such as Mosaic) should browse the Table
   of Contents.

   Lynx and other text based browsers may browse a text version, with all
   the equations in it and other goodies may read the Text Table of
   Contents.

   A dvi version for on-screen browsing is also available. This version
   has very small pages, so that users with 14" monitors may set a high
   magnification value in xdvi and still be able to read each page
   without scrolling. This file is 228 KB long .

   A printer ready dvi version. Users with 17" monitors or good eyesight
   may prefer this version. This file is 221 KB long.
     _________________________________________________________________
archive-name: sci-math-faq/tableofcontents 
Last-modified: Dec 8, 1994
Version: 6.1

The NEW FAQ is out, and will be posted by instalments. Here is the table
of contents.

   
Contents

     * Contents
     * Introduction
	  + Why a list of Frequently Asked Questions?
	  + Frequently Asked Questions in Mathematics?
     * Fermat's Last Theorem
	  + History of Fermat's Last Theorem
	  + What is the current status of FLT?
	  + Wiles' line of attack
	  + If not, then what?
	       o What has been proved
	       o Conjectures
	  + Did Fermat prove this theorem?
     * Prime Numbers
	  + Largest known Mersenne prime
	  + Largest known prime
	  + Largest known twin primes
	  + Largest Fermat number with known factorization
	  + Algorithms to factor integer numbers
	  + List of record numbers
	  + What is the current status on Mersenne primes?
	  + Formulae to compute prime numbers
     * What are numbers?
	  + Introduction
	  + Construction of the Number System
	  + Construction of N
	  + Construction of Z
	  + Construction of Q
	  + Construction of R
	  + Construction of C
	  + Rounding things up
	  + What's next?
     * Special Numbers and Functions
	  + How to compute digits of pi ?
	  + Indiana bill sets the value of pi to 3
	  + Euler's formula: e^(i pi) = -1 
	  + What is 0^0 
	  + Why is 0.9999... = 1 ?
	  + Name for f(x)^(f(x)) = x 
     * Unsolved Problems
	  + Does there exist a number that is perfect and odd?
	  + Collatz Problem
     * Symbolic Computation Packages
     * Fields Medal
	  + Historical Introduction
	  + Table of Awardees
     * Famous Problems in Mathematics
	  + The Four Colour Theorem
	  + The Trisection of an Angle
     * Mathematical Games
	  + The Monty Hall problem
	  + Master Mind
     * Projective plane of order 10
     * The Axiom of Choice
	  + Relevance of the Axiom of Choice
	  + Cutting a sphere into pieces of larger volume
     * The Continuum Hypothesis
     * Theory of quaternionic analytic functions
     * Mathematical Oddities
	  + Erdos Number
	  + Why is there no Nobel in mathematics? 
	  + Who is N. Bourbaki?
     * Which are the 23 Hilbert Problems?
     * Formulas of General Interest
	  + How to determine the day of the week, given the month, day
	    and year
	  + Formula for the Surface Area of a sphere in Euclidean N
	    -Space
	  + Formula to compute compound interest.
     * General Bibliography
     * The Sci.Math FAQ Team
     * Copyright Notice
     * About this document ... 
       
Welcome to sci.math.research.  Here is our charter.

    CHARTER: This newsgroup is a forum for the discussion of
	 current mathematical research.

You are encouraged to post announcements of
   - mathematics conferences
   - preprints available, electronically or physically
   - new mathematics journals
   - online services for distribution of preprints
   - availability of new mathematical software

We discourage requests for
   - a particular complicated equation to be solved symbolically
   - a particular function to be integrated symbolically
   - solutions of problems at an undergraduate level
   - elementary combinatorial algorithms
   - information about computer software

The moderators of this group are

     Dan Grayson 
     Greg Kuperberg 

			   IMPORTANT HINT

     This newsgroup is moderated, but you should post original articles and
followups in the usual way.  The news reading software automatically forwards
submitted articles to the moderator.  The moderator forwards accepted articles
to the rest of the net, and returns rejected articles to the authors.

     If you wish to communicate with the moderator, one good way to do it is
to post an article which begins with "Dear Moderator".  Or you may send email
to the moderator at sci-math-research-request@uiuc.edu.  If you have any
comments about the moderation, please submit them.

				HINTS

     When posting a followup, include only enough previously posted text to
remind the reader of the topic unambiguously.  

     Crossposting to other moderated newsgroups is discouraged, because the
moderator has no criteria for judging the acceptability of the article to the
other newsgroups.  It is better to post the article separately to other
moderated newsgroups, so the other moderator can receive and judge the
article.

     Crossposting to other unmoderated newsgroups is discouraged, too,
because it leads to parallel and hence wasteful discussions, especially if
the other newsgroup is sci.math.  Nevertheless, a crossposted article will be
accepted, but please be patient - the article will not appear in the other
newsgroups until it has been moderated.  The news software detects the
presence of a moderated group in the list of newsgroups, sends the article to
the moderator, and does not arrange for it to be posted to the other
newgsroups.  That job is left to the moderator, sigh.

     If you wish replies to come back to by email, say so, or, better,
include a header line consisting of "Followup-to: poster".  Including such a
header line will save the mdoerator the bother of composing it manually.

			OTHER RELATED NEWSGROUPS

	If you are considering posting to sci.math.research, it's possible
that the article is better suited to another moderated research group such
as:

comp.graphics.research  Highly technical computer graphics discussion.
sci.crypt.research      Cryptography, cryptanalysis, and related issues.
sci.econ.research       Research in all fields of economics.
sci.physics.research    Current physics research.

	or to one of the following unmoderated groups:

comp.graphics.algorithms  Algorithms used in producing computer graphics.
comp.theory               Theoretical Computer Science.
comp.theory.cell-automata Discussion of all aspects of cellular automata.
comp.theory.dynamic-sys   Ergodic Theory and Dynamical Systems.
sci.crypt                 Different methods of data en/decryption.
sci.econ                  The science of economics.
sci.fractals              Objects of non-integral dimension and other chaos.
sci.logic                 Logic -- math, philosophy & computational aspects.
sci.math                  Mathematical discussions and pursuits.
sci.math.num-analysis     Numerical Analysis.
sci.math.symbolic         Symbolic algebra discussion.
sci.nonlinear             Chaotic systems and other nonlinear scientific study.
sci.op-research           Research, teaching & application of operations research.
sci.physics               Physical laws, properties, etc.
sci.physics.particle      Particle physics discussions.
sci.stat.math             Statistics from a strictly mathematical viewpoint.

			       ARCHIVES

     Archives of all articles posted to sci.math.research are maintained
by Michael Boardman and Lake Forest College is available via gopher to
math.lfc.edu, under the heading
	- Mathematics Related Items
and its subheading
	- Archive of sci.math.research USENET newsgroup.
Alternatively, it can be obtained under the URL 
	gopher://math.lfc.edu/11/MathRelItems/scimathArchive
using a world wide web server such as Mosaic.  It is indexed for fast
retrieval of articles based on words within the text of the articles.

			       POINTERS

A rapidly expanding collection of electronic preprint servers, electronic
journals, lists of e-mail addresses, and other information is available by
anonymous ftp, gopher, and especially World-Wide Web.  One noteworthy Web
page has the URL:
	http://www.math.psu.edu/OtherMath.html
This Web page is a large list of pointers to information of interest to
mathematicians.

Another important resource is the computer "e-math.ams.org",
which at the moment is a combined WWW, gopher, anonymous ftp, and login
facility of the American Mathematical Society.  Among other things,
e-math.ams.org has information about conferences and job openings, a
directory of members of the AMS, information about Mathematical
Reviews, TeX software, and the AMS Preprint and Abstract server.

Why is 0.9999... = 1 ?

   
   
   In modern mathematics, the string of symbols 0.9999... is understood
   to be a shorthand for ``the infinite sum 9/10 + 9/100 + 9/1000 + ...
   ''. This in turn is shorthand for ``the limit of the sequence of real
   numbers 9/10 , 9/10 + 9/100 , 9/10 + 9/100 + 9/1000, ... ''. Using the
   well-known epsilon-delta definition of the limit (you can find it in
   any of the given references on analysis), one can easily show that
   this limit is 1 . The statement that 0.9999... = 1 is simply an
   abbreviation of this fact.
   
   0.9999... = sum_(n = 1)^(oo) (9)/(10^n) = lim_(m --> oo) sum_(n = 1)^m
   (9)/(10^n)
   
   Choose varepsilon > 0 . Suppose delta = 1/- log_(10) varepsilon , thus
   varepsilon = 10^(-1/delta) . For every m > 1/delta we have that
   
   \left| sum_(n = 1)^m (9)/(10^n) - 1 \right| = (1)/(10^m) <
   (1)/(10^(1/delta)) = varepsilon
   
   So by the varepsilon - delta definition of the limit we have
   
   lim_(m --> oo) sum_(n = 1)^m (9)/(10^n) = 1
   
   Not formal enough? In that case you need to go back to the
   construction of the number system. After you have constructed the
   reals (Cauchy sequences are well suited for this case, see
   [Shapiro75]), you can indeed verify that the preceding proof correctly
   shows 0.9999... = 1 .
   
   An informal argument could be given by noticing that the following
   sequence of ``natural'' operations has as a consequence 0.9999... = 1
   . Therefore it's ``natural'' to assume 0.9999... = 1 .
   
   
   
   x = 0.9999... ; 10x = 10 o 0.9999... ; 10x = 9.9999... ; 10x - x =
   9.9999... - 0.9999... ; 9x = 9 ; x = 1 ;
   
   
   
   Thus 0.9999... = 1 .
   
   An even easier argument multiplies both sides of 0.3333... = 1/3 by 3
   . The result is 0.9999... = 3/3 = 1 .
   
   Another informal argument is to notice that all periodic numbers such
   as 0.46464646... are equal to the period divided over the same number
   of 9 s. Thus 0.46464646... = 46/99 . Applying the same argument to
   0.9999... = 9/9 = 1 .
   
   Although the three informal arguments might convince you that
   0.9999... = 1 , they are not complete proofs. Basically, you need to
   prove that each step on the way is allowed and is correct. They are
   also ``clumsy'' ways to prove the equality since they go around the
   bush: proving 0.9999... = 1 directly is much easier.
   
   You can even have that while you are proving it the ``clumsy'' way,
   you get proof of the result in another way. For instance, in the first
   argument the first step is showing that 0.9999... is real indeed. You
   can do this by giving the formal proof stated in the beginning of this
   FAQ question. But then you have 0.9999... = 1 as corollary. So the
   rest of the argument is irrelevant: you already proved what you wanted
   to prove.
   
   
   
   References
   
   R.V. Churchill and J.W. Brown. Complex Variables and Applications.
   5^(th) ed., McGraw-Hill, 1990.
   
   
   
   E. Hewitt and K. Stromberg. Real and Abstract Analysis.
   Springer-Verlag, Berlin, 1965.
   
   
   
   W. Rudin. Principles of Mathematical Analysis. McGraw-Hill, 1976.
   
   
   
   L. Shapiro. Introduction to Abstract Algebra. McGraw-Hill, 1975.
   
   
   
   This subsection of the FAQ is Copyright (c) 1994 Hans de Vreught. Send
   comments and or corrections relating to this part to
   hdev@cp.tn.tudelft.nl.
   
   
     _________________________________________________________________
   
   
   
    alopez-o@barrow.uwaterloo.ca
    Thu Dec 08 20:49:28 EST 1994


Archive-Name: sci-math-faq/specialnumbers/0to0
Last-modified: December 8, 1994
Version: 6.1




What is 0^0 

   
   
   According to some Calculus textbooks, 0^0 is an ``indeterminate
   form''. When evaluating a limit of the form 0^0 , then you need to
   know that limits of that form are called ``indeterminate forms'', and
   that you need to use a special technique such as L'Hopital's rule to
   evaluate them. Otherwise, 0^0 = 1 seems to be the most useful choice
   for 0^0 . This convention allows us to extend definitions in different
   areas of mathematics that otherwise would require treating 0 as a
   special case. Notice that 0^0 is a discontinuity of the function x^y .
   
   
   Rotando &Korn show that if f and g are real functions that vanish at
   the origin and are analytic at 0 (infinitely differentiable is not
   sufficient), then f(x)^g(x) approaches 1 as x approaches 0 from the
   right.
   
   From Concrete Mathematics p.162 (R. Graham, D. Knuth, O. Patashnik):
   
     Some textbooks leave the quantity 0^0 undefined, because the
     functions x^0 and 0^x have different limiting values when x
     decreases to 0. But this is a mistake. We must define x^0 = 1 for
     all x , if the binomial theorem is to be valid when x = 0 , y = 0 ,
     and/or x = -y . The theorem is too important to be arbitrarily
     restricted! By contrast, the function 0^x is quite unimportant.
     
   Published by Addison-Wesley, 2nd printing Dec, 1988.
   
   See also Knuth "Two notes on notation" (AMM 99 no. 5 (May 1992),
   403-422).
   
   On a discussion of the use of the function 0^(0^x) by an Italian
   mathematician named Guglielmo Libri.
   
     [T]he paper [33] did produce several ripples in mathematical waters
     when it originally appeared, because it stirred up a controversy
     about whether 0^0 is defined. Most mathematicians agreed that 0^0 =
     1 , but Cauchy [5, page 70] had listed 0^0 together with other
     expressions like 0/0 and oo - oo in a table of undefined forms.
     Libri's justification for the equation 0^0 = 1 was far from
     convincing, and a commentator who signed his name simply ``S'' rose
     to the attack [45]. August Mvbius [36] defended Libri, by presenting
     his former professor's reason for believing that 0^0 = 1 (basically
     a proof that lim_(x --> 0+) x^x = 1 ). Mvbius also went further and
     presented a supposed proof that lim_(x --> 0+) f(x)^(g(x)) whenever
     lim_(x --> 0+) f(x) = lim_(x --> 0+) g(x) = 0 . Of course ``S'' then
     asked [3] whether Mvbius knew about functions such as f(x) =
     e^(-1/x) and g(x) = x . (And paper [36] was quietly omitted from the
     historical record when the collected words of Mvbius were ultimately
     published.) The debate stopped there, apparently with the conclusion
     that 0^0 should be undefined.
     
     But no, no, ten thousand times no! Anybody who wants the binomial
     theorem (x + y)^n = sum_(k = 0)^n (n\choose k) x^k y^(n - k) to hold
     for at least one nonnegative integer n must believe that 0^0 = 1 ,
     for we can plug in x = 0 and y = 1 to get 1 on the left and 0^0 on
     the right.
     
     The number of mappings from the empty set to the empty set is 0^0 .
     It has to be 1.
     
     On the other hand, Cauchy had good reason to consider 0^0 as an
     undefined limiting form, in the sense that the limiting value of
     f(x)^(g(x)) is not known a priori when f(x) and g(x) approach 0
     independently. In this much stronger sense, the value of 0^0 is less
     defined than, say, the value of 0 + 0 . Both Cauchy and Libri were
     right, but Libri and his defenders did not understand why truth was
     on their side.
     
     [3] Anonymous and S... Bemerkungen zu den Aufsatze |berschrieben,
     `Beweis der Gleichung 0^0 = 1 , nach J. F. Pfaff', im zweiten Hefte
     dieses Bandes, S. 134, Journal f|r die reine und angewandte
     Mathematik, 12 (1834), 292-294.
     
     
     
     [5] OE uvres Complhtes. Augustin-Louis Cauchy. Cours d'Analyse de
     l'Ecole Royale Polytechnique (1821). Series 2, volume 3.
     
     
     
     [33] Guillaume Libri. Mimoire sur les fonctions discontinues,
     Journal f|r die reine und angewandte Mathematik, 10 (1833),
     303-316.
     
     
     
     [36] A. F. Mvbius. Beweis der Gleichung 0^0 = 1 , nach J. F. Pfaff.
     Journal f|r die reine und angewandte Mathematik,
     
     
     
     12 (1834), 134-136.
     
     [45] S... Sur la valeur de 0^0 . Journal f|r die reine und
     angewandte Mathematik 11, (1834), 272-273.
     
     
     
   
   
   
   
   References
   
   Knuth. Two notes on notation. (AMM 99 no. 5 (May 1992), 403-422).
   
   
   
   H. E. Vaughan. The expression ' 0^0 '. Mathematics Teacher 63 (1970),
   pp.111-112.
   
   
   
   Louis M. Rotando and Henry Korn. The Indeterminate Form 0^0 .
   Mathematics Magazine, Vol. 50, No. 1 (January 1977), pp. 41-42.
   
   
   
   L. J. Paige,. A note on indeterminate forms. American Mathematical
   Monthly, 61 (1954), 189-190; reprinted in the Mathematical
   Association of America's 1969 volume, Selected Papers on Calculus, pp.
   210-211.
   
   
   
   
     _________________________________________________________________
   
   
   
    alopez-o@barrow.uwaterloo.ca
    Sun Nov 20 20:45:48 EST 1994


Archive-Name: sci-math-faq/trisection
Last-modified: December 8, 1994
Version: 6.1




   
The Trisection of an Angle

   
   
   
   
   Theorem. It is not possible to split an arbitrary angle in three equal
   parts using a compass and an unmarked ruler.
   
   
   
   This problem, together with Doubling the Cube, Constructing the
   regular Heptagon and Squaring the Circle were posed by the Greeks in
   antiquity, and remained open until modern times.
   
   The solution all of them is rather inelegant from a geometric
   perspective. No geometric proof has been offered [check?], however, a
   very clever solution was found using fairly basic results from
   extension field and modern algebra.
   
   It turns out that trisecting the angle is equivalent to solving a
   cubic equation. Constructions with ruler and compass may only compute
   the solution of a limited set of such equations, even when restricted
   to integer coefficients. In particular, the equation for theta = 60
   degrees cannot be solved by ruler and compass and thus the trisection
   of the angle is not possible.
   
   
   
   
     _________________________________________________________________
   
   
   
    alopez-o@barrow.uwaterloo.ca
    Sun Nov 20 20:45:48 EST 1994


Archive-Name: sci-math-faq/fourcolour
Last-modified: December 8, 1994
Version: 6.1


The Four Colour Theorem

   
   Theorem [Four Colour Theorem] Every planar map with regions of simple
   borders can be coloured with 4 colours in such a way that no two
   regions sharing a non-zero length border have the same colour.
   
   
   
   An equivalent combinatorial interpretation is
   
   Theorem [Four Colour Theorem] Every loopless planar graph admits a
   vertex-colouring with at most four different colours.
   
   
   
   This theorem was proved with the aid of a computer in 1976. The proof
   shows that if aprox. 1,936 basic forms of maps can be coloured with
   four colours, then any given map can be coloured with four colours. A
   computer program coloured these basic forms. So far nobody has been
   able to prove it without using a computer. In principle it is possible
   to emulate the computer proof by hand computations.
   
   The number of basic forms, or configurations has been reduced to 633.
   The basic trust of the known proofs is to show that none of those
   basic configurations may appear in a minimal counterexample to the
   Four Colour Theorem because if one appeared, it could be replaced by
   something smaller, contradicting minimality.
   
   Then it is shown that all minimal counterexamples must contain one of
   the basic configurations.
   
   A recent simplification of the Four Colour Theorem proof, by
   Robertson, Sanders, Seymour and Thomas, greatly simplifies the
   original proof by Appel and Haken.
   
   Further, the set of computer programs involved, will be made available
   via anonymous ftp for all to check.
   
   If correct, this new proof should dispel all criticisms of the
   computer assisted proof.
   
   
   
   References
   
   K. Appel and W. Haken. Every planar map is four colorable. Bulletin of
   the American Mathematical Society, vol. 82, 1976 pp.711-712.
   
   
   
   K. Appel and W. Haken. Every planar map is four colorable. Illinois
   Journal of Mathematics, vol. 21, 1977, pp. 429-567.
   
   
   
   N. Robertson, D. Sanders, P. Seymour, R. Thomas The Four Colour
   Theorem Preprint, February 1994.
   
   
   
   The Four Color Theorem: Assault and Conquest T. Saaty and Paul Kainen.
   McGraw-Hill, 1977. Reprinted by Dover Publications 1986.
   
   
   
   
     _________________________________________________________________
   
   
   
    alopez-o@barrow.uwaterloo.ca
    Sun Nov 20 20:45:48 EST 1994


Archive-Name: sci-math-faq/AC/relevance
Last-modified: December 8, 1994
Version: 6.1




Relevance of the Axiom of Choice

   
   
   THE AXIOM OF CHOICE
   
   There are many equivalent statements of the Axiom of Choice. The
   following version gave rise to its name:
   
     For any set X there is a function f , with domain X ; (0) , so that
     f(x) is a member of x for every nonempty x in X .
     
   Such an f is called a "choice function" on X . [Note that X ; (0)
   means X with the empty set removed. Also note that in Zermelo-Fraenkel
   set theory all mathematical objects are sets so each member of X is
   itself a set.]
   
   The Axiom of Choice (AC) is one of the most discussed axioms of
   mathematics, perhaps second only to Euclid's parallel postulate. The
   axioms of set theory provide a foundation for modern mathematics in
   the same way that Euclid's five postulates provided a foundation for
   Euclidean geometry, and the questions surrounding AC are the same as
   the questions that surrounded Euclid's Parallel Postulate:
    1. Can it be derived from the other axioms?
    2. Is it consistent with the other axioms?
    3. Should we accept it as an axiom?
       
   For many sets, including any finite set, the first six axioms of set
   theory (abbreviated ZF) are enough to guarantee the existence of a
   choice function but there do exist sets for which AC is required to
   show the existence of a choice function. The existence of such sets
   was proved in 1963 by Paul Cohen. This means that AC cannot be derived
   from the other six axioms; in other words "AC is independent of ZF."
   This answers question [1] posed above.
   
   The question of whether AC is consistent with the other axioms
   (question [2] above) was answered by Goedel in 1938. Goedel showed
   that if the other axioms are consistent then AC is consistent with
   them. This is a "relative consistency" proof which is the best we can
   hope for because of Goedel's Second Incompleteness Theorem.
   
   The third question, "Should we accept it as an axiom?", moves us into
   the realm of philosophy. Today there are three major schools of
   thought concerning the use of AC:
    1. Accept it as an axiom and use it without hesitation.
    2. Accept it as an axiom but use it only when you cannot find a proof
       without it.
    3. AC is unacceptable.
       
   Most mathematicians today belong to school A. Mathematicians who are
   in school B are usually there because of a belief in Occam's Razor
   (use as few assumptions as possible when explaining something) or an
   interest in metamathematics. There are a growing number of people
   moving to school C, especially computer scientists who work on
   automated reasoning using constructive type theories.
   
   Underlying the schools of thought about the use of AC are views about
   truth and the nature of mathematical objects. Three major views are
   platonism, constructivism, and formalism.
   
   Platonism
   
   A platonist believes that mathematical objects exist independent of
   the human mind, and a mathematical statement, such as AC, is
   objectively either true or false. A platonist accepts AC only if it is
   objectively true, and probably falls into school A or C depending on
   her belief. If she isn't sure about AC's truth then she may be in
   school B so that once she finds out the truth about AC she will know
   which theorems are true.
   
   Constructivism
   
   A constructivist believes that the only acceptable mathematical
   objects are ones that can be constructed by the human mind, and the
   only acceptable proofs are constructive proofs. Since AC gives no
   method for constructing a choice set constructivists belong to school
   C.
   
   Formalism
   
   A formalist believes that mathematics is strictly symbol manipulation
   and any consistent theory is reasonable to study. For a formalist the
   notion of truth is confined to the context of mathematical models,
   e.g., a formalist would say "The parallel postulate is false in
   Riemannian geometry." but she wouldn't say "The parallel postulate is
   false." A formalist will probably not allign herself with any school.
   She will comfortably switch between A, B, and C depending on her
   current interests.
   
   So: Should you accept the Axiom of Choice? Here are some arguments for
   and against it.
   
   Against
   
     * It's not as simple, aesthetically pleasing, and intuitive as the
       other axioms.
     * It is equivalent to many statements which are not intuitive such
       as "Every set can be well ordered." How, for example, would you
       well order the reals?
     * With it you can derive non-intuitive results, such as the
       existence of a discontinuous additive function, the existence of a
       non-measurable set of reals, and the Banach-Tarski Paradox (see
       the next section of the sci.math FAQ).
     * It is nonconstructive - it conjures up a set without providing any
       sort of procedure for its construction.
       
   
   
   For
   
   The acceptance of AC is based on the belief that our intuition about
   finite sets can be extended to infinite sets. The main argument for
   accepting it is that it is useful. Many important, intuitively
   plausible theorems are equivalent to it or depend on it. For example
   these statements are equivalent to AC:
     * Every vector space has a basis.
     * Trichotomy of Cardinals: For any cardinals k and l , either k < l
       or k = l or k > l .
     * Tychonoff's Theorem: The product of compact spaces is compact in
       the product topology.
     * Zorn's Lemma: Every nonempty partially ordered set P in which each
       chain has an upper bound in P has a maximal element.
       
   And these statements depend on AC (i.e., they cannot be proved in ZF
   without AC):
     * The union of countably many countable sets is countable.
     * Every infinite set has a denumerable subset.
     * The Loewenheim-Skolem Theorem: Any first-order theory which has a
       model has a denumerable model.
     * The Baire Category Theorem: The reals are not the union of
       countably many nowhere dense sets (i.e., the reals are not
       meager).
     * The Ultrafilter Theorem: Every Boolean algebra has an ultrafilter
       on it.
       
   
   
   Alternatives to AC
   
     * Accept only a weak form of AC such as the Denumerable Axiom of
       Choice (every denumerable set has a choice function) or the Axiom
       of Dependent Choice.
     * Accept an axiom that implies AC such as the Axiom of
       Constructibility ( V = L ) or the Generalized Continuum Hypothesis
       (GCH).
     * Adopt AC as a logical axiom (Hilbert suggested this with his
       epsilon axiom). If set theory is done in such a logical formal
       system the Axiom of Choice will be a theorem.
     * Accept a contradictory axiom such as the Axiom of Determinacy.
     * Use a completely different framework for mathematics such as
       Category Theory. Note that within the framework of Category Theory
       Tychonoff's Theorem can be proved without AC (Johnstone, 1981).
       
   
   
   Test Yourself: When is AC necessary?
   
   If you are working in Zermelo-Fraenkel set theory without the Axiom of
   Choice, can you choose an element from...
    1. a finite set?
    2. an infinite set?
    3. each member of an infinite set of singletons (i.e., one-element
       sets)?
    4. each member of an infinite set of pairs of shoes?
    5. each member of inifinite set of pairs of socks?
    6. each member of a finite set of sets if each of the members is
       infinite?
    7. each member of an infinite set of sets if each of the members is
       infinite?
    8. each member of a denumerable set of sets if each of the members is
       infinite?
    9. each member of an infinite set of sets of rationals?
   10. each member of a denumerable set of sets if each of the members is
       denumberable?
   11. each member of an infinite set of sets if each of the members is
       finite?
   12. each member of an infinite set of finite sets of reals?
   13. each member of an infinite set of sets of reals?
   14. each member of an infinite set of two-element sets whose members
       are sets of reals?
       
   The answers to these questions with explanations are accessible
   through http://www.jazzie.com/ii/math/index.html
   
   
   
   References
   
   Benacerraf, Paul and Putnam, Hilary. "Philosophy of Mathematics:
   Selected Readings, 2nd edition." Cambridge University Press, 1983.
   
   Dauben, Joseph Warren. "Georg Cantor: His Mathematics and Philosophy
   of the Infinite." Princeton University Press, 1979.
   
   A. Fraenkel, Y. Bar-Hillel, and A. Levy with van Dalen, Dirk.
   "Foundations of Set Theory, Second Revised Edition." North-Holland,
   1973.
   
   Johnstone, Peter T. "Tychonoff's Theorem without the Axiom of Choice."
   Fundamenta Mathematica 113: 21-35, 1981.
   
   Leisenring, Albert C. "Mathematical Logic and Hilbert's
   Epsilon-Symbol." Gordon and Breach, 1969.
   
   Maddy, "Believing the Axioms, I", J. Symb. Logic, v. 53, no. 2, June
   1988, pp. 490-500, and "Believing the Axioms II" in v.53, no. 3.
   
   Moore, Gregory H. "Zermelo's Axiom of Choice: Its Origins,
   Development, and Influence." Springer-Verlag, 1982.
   
   Rubin, Herman and Rubin, Jean E. "Equivalents of the Axiom of Choice
   II." North-Holland, 1985.
   
   This section of the FAQ is Copyright (c) 1994 Nancy McGough. Send
   comments and or corrections relating to this part to nancym@ii.com.
   The most up to date version of this section of the sci.math FAQ is
   accesible through http://www.jazzie.com/ii/math/index.html
   
   
     _________________________________________________________________
   
   This section of the FAQ is Copyright (c) 1994 Nancy McGough. Send
   comments and or corrections relating to this part to nancym@ii.com.
   The most up to date version of this section of the sci.math FAQ is
   accesible through http://www.jazzie.com/ii/math/index.html

Archive-Name: sci-math-faq/bibliography
Last-modified: December 8, 1994
Version: 6.1




   
   
   
   
			    GENERAL BIBLIOGRAPHY
				       
   
   
   The books listed in this section are both good introductory books to
   mathematics, and classic references for the advanced mathematician.
   
     * Algebra
       
       Lang, Serge. Algebra
       
       Birkhoff, McLane, Algebra
       
     * Analysis
       
       Rudin
       
       Hewitt, Stromberg
       
     * Geometry
       
       David Hilbert. Foundations of Geometry 2nd English edition, tr. by
       Leo Unger, publ. by Open Court, 1971.
       
     * Number Theory
       
       Hardy, Littlewood.
       
     * Morris Kline Mathematical Thought from Ancient to Modern Times
       
     *
       
       Guillemin, Victor and Alan Pollack: Differential Topology. Spivak,
       Michael: A Comprehensive Introduction to Differential Geometry,
       Vol. I
       
     * Morgan, Frank: Riemannian Geometry: A Beginner's Guide
       
     * Greenberg, Martin and (?) Harper: Algebraic Topology: An
       Introduction.
       
   
     _________________________________________________________________
   
   
   
    alopez-o@barrow.uwaterloo.ca
    Sun Nov 20 20:45:48 EST 1994


Archive-Name: sci-math-faq/AC/cuttingSphere
Last-modified: December 8, 1994
Version: 6.1


Cutting a sphere into pieces of larger volume

   
   
   Is it possible to cut a sphere into a finite number of pieces and
   reassemble into a solid of twice the volume?
   
   This question has many variants and it is best answered explicitly.
   
   Given two polygons of the same area, is it always possible to dissect
   one into a finite number of pieces which can be reassembled into a
   replica of the other?
   
   Dissection theory is extensive. In such questions one needs to specify
   
   
     * What is a ``piece"? (polygon? Topological disk? Borel-set?
       Lebesgue-measurable set? Arbitrary?)
       
     * How many pieces are permitted (finitely many? countably?
       uncountably?)
       
     * What motions are allowed in ``reassembling" (translations?
       rotations? orientation-reversing maps? isometries? affine maps?
       homotheties? arbitrary continuous images? etc.)
       
     * How the pieces are permitted to be glued together. The simplest
       notion is that they must be disjoint. If the pieces are polygons
       [or any piece with a nice boundary] you can permit them to be
       glued along their boundaries, ie the interiors of the pieces
       disjoint, and their union is the desired figure.
       
   
   
   Some dissection results
   
     * We are permitted to cut into finitely many polygons, to translate
       and rotate the pieces, and to glue along boundaries; then yes, any
       two equal-area polygons are equi-decomposable.
       
       This theorem was proven by Bolyai and Gerwien independently, and
       has undoubtedly been independently rediscovered many times. I
       would not be surprised if the Greeks knew this.
       
       The Hadwiger-Glur theorem implies that any two equal-area polygons
       are equi-decomposable using only translations and rotations by 180
       degrees.
       
     *
       
       Theorem [Hadwiger-Glur, 1951] Two equal-area polygons P,Q are
       equi-decomposable by translations only, iff we have equality of
       these two functions: phi_P() = phi_Q()
       
       Here, for each direction v (ie, each vector on the unit circle in
       the plane), let phi_P(v) be the sum of the lengths of the edges of
       P which are perpendicular to v , where for such an edge, its
       length is positive if v is an outward normal to the edge and is
       negative if v is an inward normal to the edge.
       
     * In dimension 3, the famous ``Hilbert's third problem" is:
       
     If P and Q are two polyhedra of equal volume, are they
     equi-decomposable by means of translations and rotations, by cutting
     into finitely many sub-polyhedra, and gluing along boundaries?
   
       
       The answer is NO and was proven by Dehn in 1900, just a few months
       after the problem was posed. (Ueber raumgleiche polyeder,
       Goettinger Nachrichten 1900, 345-354). It was the first of
       Hilbert's problems to be solved. The proof is nontrivial but does
       not use the axiom of choice.
       
       
       
       References
       
       Hilbert's Third Problem. V.G. Boltianskii. Wiley 1978.
       
       
       
     * Using the axiom of choice on non-countable sets, you can prove
       that a solid sphere can be dissected into a finite number of
       pieces that can be reassembled to two solid spheres, each of same
       volume of the original. No more than nine pieces are needed.
       
       The minimum possible number of pieces is five. (It's quite easy to
       show that four will not suffice). There is a particular dissection
       in which one of the five pieces is the single center point of the
       original sphere, and the other four pieces A , A' , B , B' are
       such that A is congruent to A' and B is congruent to B' . [See
       Wagon's book].
       
       This construction is known as the Banach-Tarski paradox or the
       Banach-Tarski-Hausdorff paradox (Hausdorff did an early version
       of it). The ``pieces" here are non-measurable sets, and they are
       assembled disjointly (they are not glued together along a
       boundary, unlike the situation in Bolyai's thm.) An excellent book
       on Banach-Tarski is:
       
       The Banach-Tarski Paradox. Stan Wagon. Cambridge University Press,
       985
       
       
       
       Robert M. French. The Banach-Tarski theorem. The Mathematical
       Intelligencer, 10 (1988) 21-28.
       
       
       
       The pieces are not (Lebesgue) measurable, since measure is
       preserved by rigid motion. Since the pieces are non-measurable,
       they do not have reasonable boundaries. For example, it is likely
       that each piece's topological-boundary is the entire ball.
       
       The full Banach-Tarski paradox is stronger than just doubling the
       ball. It states:
       
     * Any two bounded subsets (of 3-space) with non-empty interior, are
       equi-decomposable by translations and rotations.
       
       This is usually illustrated by observing that a pea can be cut up
       into finitely pieces and reassembled into the Earth.
       
       The easiest decomposition ``paradox" was observed first by
       Hausdorff:
       
     * The unit interval can be cut up into countably many pieces which,
       by translation only, can be reassembled into the interval of
       length 2.
       
       This result is, nowadays, trivial, and is the standard example of
       a non-measurable set, taught in a beginning graduate class on
       measure theory.
     *
       
       Theorem. There is a finite collection of disjoint open sets in the
       unit cube in R^3 which can be moved by isometries to a finite
       collection of disjoint open sets whose union is dense in the cube
       of size 2 in R^3.
       
       This result is by Foreman and Dougherty.
       
   
   
   
   
   References
   
   Boltyanskii. Equivalent and equidecomposable figures. in Topics in
   Mathematics published by D.C. HEATH AND CO., Boston.
   
   
   
   Dubins, Hirsch and ? Scissor Congruence American Mathematical Monthly.
   
   
   
   
   ``Banach and Tarski had hoped that the physical absurdity of this
   theorem would encourage mathematicians to discard AC. They were
   dismayed when the response of the math community was `Isn't AC great?
   How else could we get such counterintuitive results?' ''
   
   
   
   
     _________________________________________________________________
   
   
   
    alopez-o@barrow.uwaterloo.ca
    Sun Nov 20 20:45:48 EST 1994


Archive-Name: sci-math-faq/bourbaki
Last-modified: December 8, 1994
Version: 6.1

   
   
     _________________________________________________________________
   
   Next: Which are the Up: Mathematical Oddities Previous: Why is there 
     _________________________________________________________________
   
   
   
Who is N. Bourbaki?

   
   
   A group of mostly French mathematicians which began meeting in the
   1930s, aiming to write a thorough unified account of all mathematics.
   They had tremendous influence on the way math is done since. For a
   very accessible sampler see Dieudonne MATHEMATICS: THE MUSIC OF REASON
   (orig. POUR l'HONNEUR DE l'ESPRIT HUMAIN).
   
   The founding is described in Andre Weil's autobiography, titled
   something like "memoir of an apprenticeship" (orig. SOUVENIRS
   D'APPRENTISSAGE). There is a usable book BOURBAKI by J. Fang. Liliane
   Beaulieu has a book forthcoming, which you can sample in "A Parisian
   Cafe and Ten Proto-Bourbaki Meetings 1934-1935" in the MATHEMATICAL
   INTELLIGENCER 15 no.1 (1993) 27-35.
   
   
   
   
     _________________________________________________________________
   
   
   
    alopez-o@barrow.uwaterloo.ca
    Sun Nov 20 20:45:48 EST 1994


Archive-Name: sci-math-faq/compoundInterest
Last-modified: December 8, 1994
Version: 6.1

   
   
Formula to compute compound interest.

   Here's a formula which can be used in 123, Excel, Wings and Dynaplan:
   

     ------- Input this data -------------------------------
     principal amount = E9                  ( in dollars )
     Amortization Period = d10              ( in years ie 6 mon = .5 )
     Payments / year = D11                  ( 12 = monthly, 52 = weekly )
     Published Interest rate = D12          ( ie 9 % = 0.09 )
     Times per year Int calculated = d13    ( CDN mortgage use 2
					 US mortgage use 12
					 all other loans use 12 )
     ----- Calculate the proper rate of interest -----------
     e14 = Effective annual rate = EXP(D13*LN(1+(D12/D13)))-1
     e15 = Interest rate per payment = (EXP(LN(E14+1)/(D10*D11))-1)*D10*D11
     e17 = Payments = APMT(E9,E15/D11,D10*D11) ( both these functions are
		    = PMT (E9,E15/D11,D10*D11) ( identical,diff spreadsheet)
	   APMT( principal amount,interest rate per period,# periods )
	   ( this is a standard function on any true commercial spreadsheet)
	   OR use the following if done using a calculator
	 = Payments = P*I/[1-(I+1)^-T]
		    = E9*(E15/D11)/(1-((E15/D11) +1)**(-1*D10*D11))
     Total interest cost = E17*D10*D11-E9
     -- Use these formulas if you wish to generate an amortization table --
     always add up to 'Payments (e17)'
     Interest per payment  = current balance * ( E15 / D11 )
     Principal per payment = current balance - Interest per payment
     new current balance   = current balance - Principal per payment -
			     (extra payment)
	keep repeating until 'new current balance' = 0

   
   
   Derivation of Compound Interest Rate Formula
   
   Suppose you deposited a fixed payment into an interest bearing account
   at regular intervals, say monthly, at the end of each month. How much
   money would there be in the account at the end of the nth month (at
   which point you've made i payments)?
   
   Let x be the monthly interest rate as a fraction of principle.
   Let n be the amount deposited each month.
   Let p[k] be the total number of months.
   Let k be the principle after p[n] = x + ((1 + i) p[n - 1]) eq 1
   months.
   So the recursive formula is:
   
   p[n] = sum_(k = 0)^(n - 1)x (1 + i)^k
   
   This yields the summation:
   
   (1 + i)
   
   The way to solve this is to multiply through by pi p[n] = x ((1 + i)^n
   - 1) and subtract the original equation from the resulting equation.
   Observe that all terms in the summation cancel except the last term of
   the multiplied equation and the first term of the original equation:
   
   p[n] = x ((1 + i)^n - 1)/i
   
   or
   
   p
   
   Now suppose you borrow i at constant interest rate x . You make
   monthly payments of p . It turns out that this problem is identical to
   taking out a balloon loan of x (that is it's all due at the end of
   some term) and putting payments of n into a savings account. At the
   end of the term you use the principle in the savings account to pay
   off the balance of the loan. The loan and the savings account, of
   course, must be at the same interest rate. So what we want to know is:
   what monthly payment is needed so that the balance of the savings
   account will be identical to the balance of the balloon loan after n
   payments?
   
   The formula for the principal of the balloon loan at the end of the
   p[n] = p[0] (1 + i)^n th month is:
   
   p[0] (1 + i)^n = x ((1 + i)^n - 1)/i
   
   So we set this expression equal to the expression for the the savings
   account, and we get:
   
   x = p[0] (1 + i)^n i/((1 + i)^n - 1)
   
   or solving for x:
   
   (1 + i)^n
   
   If n is large enough (say greater than 5), here is an approximation
   for determining x from p , i , and n ~= -ln(ln(x/(ip)))/ln(1 + i) :
   
   ln(y - 1) ~= ln y - 1/y
   
   The above approximation is based upon the following approximation:
   
   y >= 5
   
   Which is within 2 For example, a $100000loan at 1%monthly, paying
   $1028.61 per month should be paid in 360 months. The approximation
   yields 358.9 payments.
   
   If this were your 30 year mortgage and you were paying $1028.61 per
   month and you wanted to see the effect of paying $1050per month, the
   approximation tells you that it would be paid off in 303.5 months (25
   years and 3.5 months). If you stick 304 months into the equation for x
   , you get $1051.04, so it is fairly close. This approximation does not
   work, though, for very small interest rates or for a small number of
   payments. The rule is to get a rough idea first of what (1 + i)^n is.
   If that is greater than 5, the approximation works pretty well. In the
   examples given, (1 + i)^n is about 36.
   
   Finding i given n , x , and p is not as easy. If i is less than 5%per
   payment period, the following equation approximately holds for i:
   
   i = -(1/n) ln(1 - ip/x)
   
   There is no direct solution to this, but you can do it by
   Newton-Raphson approximation. Begin with a guess, i[0]. Then apply:
   
   i[k + 1] = i[k] - (x(1 - i[k]p/x) (ni[k] + ln(1 - i[k]p/x)))/(xn(1 -
   i[k]p/x) - p)
   
   You must start with i too big, because the equation for i has a
   solution at i = 0 , and that's not the one you want to end up with.
   
   Example: Let the loan be for p = $10000 , x = $50 per week for 5 years
   ( n = 260 ). Let i[0] = 20 %per annum or 0.3846%per week. Since i must
   be a fraction rather than a percent, i[0] = 0.003846 . Then, applying
   eq 11:
   
   
   
   i[1] = 0.003077 ; i[2] = 0.002479 ; i[3] = 0.002185 ; i[4] = 0.002118
   ; i[5] = 0.002115 ;
   
   
   
   The series is clearly beginning to converge here.
   
   To get i[5] as an annual percentage rate, multiply by 52 weeks in a
   year and then by 100%, so i[5] = 10.997 %per annum. Substituting i[5]
   back into eq 7, we get x = $50.04 , so it works pretty well.
   
   
   
   References
   
   The theory of interest. Stephen G. Kellison. Homewood, Ill., R. D.
   Irwin, 1970.
   
   
   
   
   
   
     _________________________________________________________________
   
   
   
    alopez-o@barrow.uwaterloo.ca
    Sun Nov 20 20:45:48 EST 1994


Archive-Name: sci-math-faq/specialnumbers/computePi
Last-modified: December 8, 1994
Version: 6.1

How to compute digits of pi ?

   
   
   Symbolic Computation software such as Maple or Mathematica can compute
   10,000 digits of pi in a blink, and another 20,000-1,000,000 digits
   overnight (range depends on hardware platform).
   
   It is possible to retrieve 1.25+ million digits of pi via anonymous
   ftp from the site wuarchive.wustl.edu, in the files pi.doc.Z and
   pi.dat.Z which reside in subdirectory doc/misc/pi. New York's
   Chudnovsky brothers have computed 2 billion digits of pi on a homebrew
   computer.
   
   There are essentially 3 different methods to calculate pi to many
   decimals.
   
    1. One of the oldest is to use the power series expansion of atan(x)
       = x - x^3/3 + x^5/5 - ... together with formulas like pi =
       16*atan(1/5) - 4*atan(1/239) . This gives about 1.4 decimals per
       term.
       
    2. A second is to use formulas coming from Arithmetic-Geometric mean
       computations. A beautiful compendium of such formulas is given in
       the book pi and the AGM, (see references). They have the advantage
       of converging quadratically, i.e. you double the number of
       decimals per iteration. For instance, to obtain 1 000 000
       decimals, around 20 iterations are sufficient. The disadvantage is
       that you need FFT type multiplication to get a reasonable speed,
       and this is not so easy to program.
       
    3. A third one comes from the theory of complex multiplication of
       elliptic curves, and was discovered by S. Ramanujan. This gives a
       number of beautiful formulas, but the most useful was missed by
       Ramanujan and discovered by the Chudnovsky's. It is the following
       (slightly modified for ease of programming):
       
       Set k_1 = 545140134; i k_2 = 13591409; k_3 = 640320; k_4 =
       100100025; k_5 = 327843840; k_6 = 53360;
       
       Then pi = (k_6 sqrt(k_3))/(S) , where
       
       S = sum_(n = 0)^oo (-1)^n ((6n)!(k_2 +
       nk_1))/(n!^3(3n)!(8k_4k_5)^n)
       
       The great advantages of this formula are that
       
       1) It converges linearly, but very fast (more than 14 decimal
       digits per term).
       
       2) The way it is written, all operations to compute S can be
       programmed very simply since it only involves
       multiplication/division by single precision numbers. This is why
       the constant 8k_4k_5 appearing in the denominator has been written
       this way instead of 262537412640768000. This is how the
       Chudnovsky's have computed several billion decimals.
       
   
   
   The following 160 character C program, written by Dik T. Winter at
   CWI, computes pi to 800 decimal digits.
   

     int a=10000,b,c=2800,d,e,f[2801],g;main(){for(;b-c;)f[b++]=a/5;
     for(;d=0,g=c*2;c-=14,printf("%.4d",e+d/a),e=d%a)for(b=c;d+=f[b]*a,
     f[b]=d%--g,d/=g--,--b;d*=b);}

   
   
   
   
   References
   
   P. B. Borwein, and D. H. Bailey. Ramanujan, Modular Equations, and
   Approximations to pi American Mathematical Monthly, vol. 96, no. 3
   (March 1989), p. 201-220.
   
   
   
   J.M. Borwein and P.B. Borwein. The arithmetic-geometric mean and fast
   computation of elementary functions. SIAM Review, Vol. 26, 1984, pp.
   351-366.
   
   
   
   J.M. Borwein and P.B. Borwein. More quadratically converging
   algorithms for pi . Mathematics of Computation, Vol. 46, 1986, pp.
   247-253.
   
   
   
   Shlomo Breuer and Gideon Zwas Mathematical-educational aspects of the
   computation of pi Int. J. Math. Educ. Sci. Technol., Vol. 15, No. 2,
   1984, pp. 231-244.
   
   
   
   David Chudnovsky and Gregory Chudnovsky. The computation of classical
   constants. Columbia University, Proc. Natl. Acad. Sci. USA, Vol. 86,
   1989.
   
   
   
   Y. Kanada and Y. Tamura. Calculation of pi to 10,013,395 decimal
   places based on the Gauss-Legendre algorithm and Gauss arctangent
   relation. Computer Centre, University of Tokyo, 1983.
   
   
   
   Morris Newman and Daniel Shanks. On a sequence arising in series for
   pi . Mathematics of computation, Vol. 42, No. 165, Jan 1984, pp.
   199-217.
   
   
   
   E. Salamin. Computation of pi using arithmetic-geometric mean.
   Mathematics of Computation, Vol. 30, 1976, pp. 565-570
   
   
   
   David Singmaster. The legal values of pi . The Mathematical
   Intelligencer, Vol. 7, No. 2, 1985.
   
   
   
   Stan Wagon. Is pi normal? The Mathematical Intelligencer, Vol. 7, No.
   3, 1985.
   
   
   
   
   
   A history of pi . P. Beckman. Golem Press, CO, 1971 (fourth edition
   1977)
   
   
   
   pi and the AGM - a study in analytic number theory and computational
   complexity. J.M. Borwein and P.B. Borwein. Wiley, New York, 1987.
   
   
   
   
     _________________________________________________________________
   
   
   
    alopez-o@barrow.uwaterloo.ca
    Sun Nov 20 20:45:48 EST 1994


Archive-Name: sci-math-faq/AC/ContinuumHyp
Last-modified: December 8, 1994
Version: 6.1


   
   
			  THE CONTINUUM HYPOTHESIS
				       
   
   
   A basic reference is Godel's "What is Cantor's Continuum Problem?",
   from 1947 with a 1963 supplement, reprinted in Benacerraf and Putnam's
   collection Philosophy of Mathematics. This outlines Godel's generally
   anti-CH views, giving some "implausible" consequences of CH.
   
   "I believe that adding up all that has been said one has good reason
   to suspect that the role of the continuum problem in set theory will
   be to lead to the discovery of new axioms which will make it possible
   to disprove Cantor's conjecture."
   
   At one stage he believed he had a proof that C = aleph_2 from some new
   axioms, but this turned out to be fallacious. (See Ellentuck, "Godel's
   Square Axioms for the Continuum", Mathematische Annalen 1975.)
   
   Two people pointed me to a recent paper by Maddy: "Believing the
   Axioms", Journal of Symbolic Logic 1988 (in 2 parts). This is an
   extremely interesting paper and a lot of fun to read. A bonus is that
   it gives a non-set-theorist who knows the basics a good feeling for a
   lot of issues in contemporary set theory.
   
   Most of the first part is devoted to "plausible arguments" for or
   against CH: how it stands relative to both other possible axioms and
   to various set-theoretic "rules of thumb". One gets the feeling that
   the weight of the arguments is against CH, although Maddy says that
   many "younger members" of the set-theoretic community are becoming
   more sympathetic to CH than their elders. There's far too much here
   for me to be able to go into it in much detail.
   
   Some highlights from Maddy's discussion, also incorporating a few
   things that other people sent me:
   
    1. Cantor's reasons for believing CH aren't all that persuasive
       today.
    2. Godel's proof of the consistency of CH shows that CH follows from
       ZFC plus the Axiom of Constructibility ( V = L , roughly that the
       set-theoretic universe = the constructible universe). However,
       most set-theorists seem to find Constructiblity implausible and
       much too restrictive. It's an example of a "minimizing" principle,
       which tends to cut down on the number of sets admitted to one's
       universe. Apparently "maximizing" principles meet with much more
       sympathy from set theorists. Such principles are more compatible
       with not CH than with CH.
    3. If GCH is true, this implies that aleph_0 has certain unique
       properties: e.g. that it's that cardinal before which GCH is false
       and after which it is true. Some would like to believe that the
       set-theoretic universe is more "uniform" (homogeneous) than that,
       without this kind of singular occurrence. Such a "uniformity"
       principle tends to imply not GCH.
    4. Most of those who disbelieve CH think that the continuum is likely
       to have very large cardinality, rather than aleph_2 (as Godel
       seems to have suggested). Even Cohen, a professed formalist,
       argues that the power set operation is a strong operation that
       should yield sets much larger than those reached quickly by
       stepping forward through the ordinals:
       
     "This point of view regards C as an incredibly rich set given to us
     by a bold new axiom, which can never be approached by any piecemeal
     process of construction."
    5. There are also a few arguments in favour of CH, e.g. there's an
       argument that not CH is restrictive (in the sense of (2) above).
       Also, CH is much easier to force (Cohen's method) than not -CH.
       And CH is much more likely to settle various outstanding results
       than is not CH, which tends to be neutral on these results.
    6. Most large cardinal axioms (asserting the existence of cardinals
       with various properties of hugeness: these are usually derived
       either from considering the hugeness of aleph_0 compared to the
       finite cardinals and applying uniformity, or from considering the
       hugeness of V (the set-theoretic universe) relative to all sets
       and applying "reflection") don't seem to settle CH one way or the
       other.
    7. Various other axioms have some bearing. Axioms of determinacy
       restrict the class of sets of reals that might be counterexamples
       to CH. Various forcing axioms (e.g. Martin's axiom), which are
       "maximality" principles (in the sense of (2) above), imply not CH.
       The strongest (Martin's maximum) implies that C = aleph_2 . Of
       course the "truth" or otherwise of all these axioms is
       controversial.
    8. Freiling's principle about "throwing darts at the real line" is a
       seemingly very plausible principle, not involving large cardinals
       at all, from which not CH immediately follows. Freiling's paper
       (JSL 1986) is a good read. More on this at the end of this
       message.
       
   
   
   Of course I've conspicuously avoided saying anything about whether
   it's even reasonable to suppose that CH has a determinate truth-value.
   Formalists will argue that we may choose to make it come out whichever
   way we want, depending on the system we work in. On the other hand,
   the mere fact of its independence from ZFC shouldn't immediately lead
   us to this conclusion - this would be assigning ZFC a privileged
   status which it hasn't necessarily earned. Indeed, Maddy points out
   that various axioms within ZFC (notably the Axiom of Choice, and also
   Replacement) were adopted for extrinsic reasons (e.g. "usefulness") as
   well as for "intrinsic" reasons (e.g. "intuitiveness"). Further
   axioms, from which CH might be settled, might well be adopted for such
   reasons.
   
   One set-theorist correspondent said that set-theorists themselves are
   very loathe to talk about "truth" or "falsity" of such claims.
   (They're prepared to concede that 2 + 2 = 4 is true, but as soon as
   you move beyond the integers trouble starts. e.g. most were wary even
   of suggesting that the Riemann Hypothesis necessarily has a
   determinate truth-value.) On the other hand, Maddy's contemporaries
   discussed in her paper seemed quite happy to speculate about the
   "truth" or "falsity" of CH.
   
   Personally, not only do I see the integers as bedrock, but I'm also
   prepared to take any finite number of power sets before I have any
   problems. At least that far, I'm a diehard Platonist. I'm considerably
   less sure whether to be a diehard Platonist about the paradise of
   ridiculously big cardinals. But my intuition is happy with reals and
   sets of reals, so as a naive non-set-theorist I'm sticking to the
   belief that CH is determinate one way or the other. As one
   correspondent suggested, the question of the determinateness of CH is
   perhaps the single best way to separate the Platonists from the
   formalists.
   
   And is it true or false? Well, I'd always found CH somewhat
   intuitively plausible. But after reading all this, it does seem that
   the weight of evidence tend to point the other way. On the other hand,
   that's only a second-order approximation based on limited knowledge -
   more subtle and sophisticated arguments (third-order approximations)
   might begin to push things the other way. Still, if I had to lay money
   on it (God, of course, knows the right answer and could settle the
   bet), I'd feel safer with the money on not CH.
   
   I've enclosed (with permission) a brief but enlightening message from
   Bill Allen on Freiling's Axiom of Symmetry. This is a good one to run
   your intuitions by.
   
   Many thanks to Bill Allen, James Cummings, David Feldman, Torkel
   Franzen, Calvin Ostrum, Keith Ramsay, and Peter Suber. I'd be very
   interested to hear more about this subject. Let A be the set of
   functions mapping Real Numbers into countable sets of Real Numbers.
   Given a function f in A , and some arbitrary real numbers x and y , we
   see that x is in f(y) with probability 0, i.e. x is not in f(y) with
   probability 1. Similarly, y is not in f(x) with probability 1. Let AX
   be the axiom which states
   
   "for every f in A , there exist x and y such that x is not in f(y) and
   y is not in f(x) "
   
   The intuitive justification for AX is that we can find the x and y by
   choosing them at random.
   
   In ZFC, AX = not CH. proof: If CH holds, then well-order R as r_0,
   r_1, .... , r_x, ... with x < aleph_1 . Define f(r_x) as { r_y : y >=
   x } . Then f is a function which witnesses the falsity of AX.
   
   If CH fails, then let f be some member of A . Let Y be a subset of R
   of cardinality aleph_1 . Then Y is a proper subset. Let X be the union
   of all the sets f(y) with y in Y , together with Y . Then, as X is an
   aleph_1 union of countable sets, together with a single aleph_1 size
   set Y , the cardinality of X is also aleph_1 , so X is not all of R .
   Let a be in R X , so that a is not in f(y) for any y in Y . Since f(a)
   is countable, there has to be some b in Y such that b is not in f(a) .
   Thus we have shown that there must exist a and b such that a is not in
   f(b) and b is not in f(a) . So AX holds.
   
   I like Freiling's proof, since it does not invoke large cardinals or
   intense infinitary combinatorics to make the point that CH implies
   counter-intuitive propositions. Freiling has also pointed out that the
   natural extension of AX is AXL (notation mine), where AXL is AX with
   the notion of countable replaced by Lebesgue Measure zero. Freiling
   has established some interesting Fubini-type theorems using AXL.
   
   See "Axioms of Symmetry: Throwing Darts at the Real Line", by
   Freiling, Journal of Symbolic Logic, 51, pages 190-200. An extension
   of this work appears in "Some properties of large filters", by
   Freiling and Payne, in the JSL, LIII, pages 1027-1035, but Chris tells
   me he's not as fond of the latter paper as he is the former.
   
   
   
   
     _________________________________________________________________
   
   
   
    alopez-o@barrow.uwaterloo.ca
    Sun Nov 20 20:45:48 EST 1994


   
   
   
   
   
     _________________________________________________________________
   
   
   
    alopez-o@barrow.uwaterloo.ca
    Thu Dec 08 20:49:28 EST 1994

Archive-Name: sci-math-faq/dayWeek
Last-modified: December 8, 1994
Version: 6.1


How to determine the day of the week, given the month, day and year

   
   
   First a brief explanation: In the Gregorian Calendar, over a period of
   four hundred years, there are 97 leap years and 303 normal years. Each
   normal year, the day of January 1 advances by one; for each leap year
   it advances by two.
   
   303 + 97 + 97 = 497 = 7 * 71
   
   As a result, January 1 year N occurs on the same day of the week as
   January 1 year N + 4004 . Because the leap year pattern also recurs
   with a four hundred year cycle, a simple table of four hundred
   elements, and single modulus, suffices to determine the day of the
   week (in the Gregorian Calendar), and does it much faster than all the
   other algorithms proposed. Also, each element takes (in principle)
   only three bits; the entire table thus takes only 1200 bits, or 300
   bytes; on many computers this will be less than the instructions to do
   all the complicated calculations proposed for the other algorithms.
   
   Incidental note: Because 7 does not divide 400, January 1 occurs more
   frequently on some days than others! Trick your friends! In a cycle of
   400 years, January 1 and March 1 occur on the following days with the
   following frequencies:
   

	   Sun      Mon     Tue     Wed     Thu     Fri     Sat
    Jan 1: 58       56      58      57      57      58      56
    Mar 1: 58       56      58      56      58      57      57

   
   
   Of interest is that (contrary to most initial guesses) the occurrence
   is not maximally flat.
   
   The Gregorian calendar was introduced in 1582 in parts of Europe; it
   was adopted in 1752 in Great Britain and its colonies, and on various
   dates in other countries. It replaced the Julian Calendar which has a
   four-year cycle of leap years; after four years January 1 has advanced
   by five days. Since 5 is relatively prime to 7, a table of 4 * 7 = 28
   elements is necessary for the Julian Calendar.
   
   There is still a 3 day over 10,000 years error which the Gregorian
   calendar does not take into account. At some time such a correction
   will have to be done but your software will probably not last that
   long!
   
   Here is a standard method suitable for mental computation:
   
    1. Take the last two digits of the year.
    2. Divide by 4, discarding any fraction.
    3. Add the day of the month.
    4. Add the month's key value: JFM AMJ JAS OND 144 025 036 146
    5. Subtract 1 for January or February of a leap year.
    6. For a Gregorian date, add 0 for 1900's, 6 for 2000's, 4 for
       1700's, 2 for 1800's; for other years, add or subtract multiples
       of 400.
    7. For a Julian date, add 1 for 1700's, and 1 for every additional
       century you go back.
    8. Add the last two digits of the year.
    9. Divide by 7 and take the remainder.
       
   
   
   Now 1 is Sunday, the first day of the week, 2 is Monday, and so on.
   
   The following formula, which is for the Gregorian calendar only, may
   be more convenient for computer programming. Note that in some
   programming languages the remainder operation can yield a negative
   result if given a negative operand, so mod 7 may not translate to a
   simple remainder. W = (k + floor(2.6m - 0.2) - 2C + Y + floor(Y/4) +
   floor(C/4)) mod 7 where floor() denotes the integer floor function,
   k is day (1 to 31)
   m is month (1 = March, ..., 10 = December, 11 = Jan, 12 = Feb) Treat
   Jan &Feb as months of the preceding year
   C is century (1987 has C = 19)
   Y is year (1987 has Y = 87 except Y = 86 for Jan &Feb)
   W is week day (0 = Sunday, ..., 6 = Saturday)
   Here the century and 400 year corrections are built into the formula.
   The floor(2.6m - 0.2) term relates to the repetitive pattern that the
   30-day months show when March is taken as the first month.
   
   The following short C program works for a restricted range
   

dow(m,d,y){y-=m<3;return(y+y/4-y/100+y/400+"-bed=pen+mad."[m]+d)%7;}

   
   
   The program appeared was posted by sakamoto@sm.sony.co.jp (Tomohiko
   Sakamoto) on comp.lang.c on March 10th, 1993.
   
   A good mnemonic rule to help on the computation of the day of the week
   is as follows. In any given year the following days come on the same
   day of the week:
   

4/4
6/6
8/8
10/10
12/12

   
   
   to remember the next for remember that I work from 9-5 at a 7-11 so
   

9/5
5/9
7/11
11/7

   
   
   and the last day of Feb.
   
   This year they come on a Monday. Every year this advances one (so next
   year they come on a Tuesday) other than leap-years which advance 2.
   Thus every 4 years it advances 5 days (there is one more minor
   correction at the century mark).
   
   Even ignoring the pattern over for a period of years this is still
   useful since you can generally figure out what day of the week a given
   date is on faster than someone else can look it up with a calender if
   the calender is not right there. (A useful skill that.)
   
   
   
   References
   
   Winning Ways for your mathematical plays. Guy Conway and Elwyn
   Berlekamp London ; Toronto : Academic Press, 1982.
   
   
   
   Mathematical Carnival. Martin Gardner. New York : Knopf, c1975.
   
   
   
   Elementary Number Theory and its applications. Kenneth Rosen. Reading,
   Mass. ; Don Mills, Ont. : Addison-Wesley Pub. Co., c1993. p. 156.
   
   
   
   Michael Keith and Tom Craver. The Ultimate Perpetual Calendar? Journal
   of Recreational Mathematics, 22:4, pp. 280-282, 19
   
   
   
   
     _________________________________________________________________
   
   
   
    alopez-o@barrow.uwaterloo.ca
    Sun Nov 20 20:45:48 EST 1994

Archive-Name: sci-math-faq/erdos 
Last-modified: December 8, 1994
Version: 6.1


Erdos Number

   
   Form an undirected graph where the vertices are academics, and an edge
   connects academic X to academic Y if X has written a paper with Y .
   The Erdos number of X is the length of the shortest path in this graph
   connecting X with Erdos.
   
   Erdos has Erdos number 0. Co-authors of Erdos have Erdos number 1.
   Einstein has Erdos number 2, since he wrote a paper with Ernst Straus,
   and Straus wrote many papers with Erdos.
   
   The Extended Erdos Number applies to co-authors of Erdos. For People
   who have authored more than one paper with Erdos, their Erdos number
   is defined to be 1/# papers-co-authored. Ron Graham has the smallest,
   non-zero, Erdos number.
   
   Why people care about it?
   
   Nobody seems to have a reasonable answer...
   
   Who is Paul Erdos?
   
   Paul Erdos is an Hungarian mathematician. He obtained his PhD from the
   University of Manchester and has spent most of his efforts tackling
   "small" problems and conjectures related to graph theory,
   combinatorics, geometry and number theory.
   
   He is one of the most prolific publishers of papers; and is also and
   indefatigable traveller.
   
   
   
   References
   
   Caspar Goffman. And what is your Erdos number? American Mathematical
   Monthly, v. 76 (1969), p. 791.
   
   
   
   
     _________________________________________________________________
   
   
   
    alopez-o@barrow.uwaterloo.ca
    Sun Nov 20 20:45:48 EST 1994

Archive-Name: sci-math-faq/specialnumbers/eulerFormula
Last-modified: December 8, 1994
Version: 6.1


Euler's formula: e^(i pi) = -1 , What does it mean and why is it equal to -1

   
   
   The definition and domain of exponentiation has been changed several
   times. The original operation x^y was only defined when y was a
   positive integer. The domain of the operation of exponentation has
   been extended, not so much because the original definition made sense
   in the extended domain, but because there were (almost) unique ways to
   extend exponentation which preserved many of what seemed to be the
   "important" properties of the original operation. So in part, these
   definitions are only convention, motivated by reasons of aesthetics
   and utility.
   
   The original definition of exponentiation is, of course, that x^y = x
   *x * ... * x, where x is multiplied by itself y times. This is only a
   reasonable definition for y = 1, 2, 3, ... (It could be argued that it
   is reasonable when y = 0 , but that issue is taken up in a different
   part of the FAQ). This operation has a number of properties, including
   
   
    1. x^1 = x
    2. For any x , n , m , x^n x^m = x^(n + m) .
    3. If x is positive, then x^n is positive.
       
       Now, we can try to see how far we can extend the domain of
       exponentiation so that the above properties (and others) still
       hold. This naturally leads to defining the operation x^y on the
       domain x positive real; y rational, by setting x^(p/q) = the
       q^(th) root of x^p . This operation agrees with the original
       definition of exponentiation on their common domain, and also
       satisfies (1), (2) and (3). In fact, it is the unique operation on
       this domain that does so. This operation also has some other
       properties:
       
    4. If x > 1 , then x^y is an increasing function of y .
    5. If 0 < x < 1 , then x^y is a decreasing function of y .
       
       Again, we can again see how far we can extend the domain of
       exponentiation while still preserving properties (1)-(5). This
       leads naturally to the following definition of x^y on the domain x
       positive real; y real:
       
       If x > 1 , x^y is defined to be sup_q x^q , where q runs over all
       rationals less than or equal to y .
       
       If x < 1 , x^y is defined to be in f_q x^q , where q runs over all
       rationals less than or equal to y .
       
       If x = 1 , x^y is defined to be 1 .
       
       Again, this operation satisfies (1)-(5), and is in fact the only
       operation on this domain to do so.
       
       The next extension is somewhat more complicated. As can be proved
       using the methods of calculus, if we define e to be the number
       
       e = 1 + 1/1! + 1/2! + 1/3! + ... = 2.71828...
       
       it turns out that for every real number x ,
       
    6. e^x = 1 + x/1! + x^2/2! + x^3/3! + ...
       
       e^x is also denoted exp(x) . (This series always converges
       regardless of the value of x ).
       
       One can also define an operation ln(x) on the positive reals,
       which is the inverse of the operation of exponentiation by e. In
       other words, exp(ln(x)) = x for all positive x . Moreover,
       
    7. If x is positive, then x^y = exp(y ln(x)) . Because of this, the
       natural extension of exponentiation to complex exponents, seems to
       be to define
       
       exp(z) = 1 + z/1! + z^2/2! + z^3/3! + ...
       
       for all complex z (not just the reals, as before), and to define
       
       x^z = exp(z ln(x))
       
       when x is a positive real and z is complex.
       
       This is the only operation x^y on the domain x positive real, y
       complex which satisfies all of (1)-(7). Because of this and other
       reasons, it is accepted as the modern definition of
       exponentiation.
       
       From the identities
       
       sin x = x - x^3/3! + x^5/5! - x^7/7! + ...
       
       cos x = 1 - x^2/2! + x^4/4! - x^6/6! + ...
       
       which are also obtainable from calculus, one sees that, for any
       real x,
       
    8. exp(ix) = cos x + i sin x.
       
       Thus, we get Euler's famous formula
       
       e^(pi i) = -1
       
       and
       
       e^(2pi i) = e^0 = 1.
       
       One can also obtain the classical addition formulae for sine and
       cosine from (8) and (1).
       
   
   
   All of the above extensions have been restricted to a positive real
   for the base. When the base x is not a positive real, it is not as
   clear-cut how to extend the definition of exponentiation. For example,
   (-1)^(1/2) could well be i or -i, (-1)^(1/3) could be -1 , 1/2 +
   sqrt(3)i/2 , or 1/2 - sqrt(3)i/2 , and so on. Some values of x and y
   give infinitely many candidates for x^y , all equally plausible. And
   of course x = 0 has its own special problems. These problems can all
   be traced to the fact that the exp function is not injective on the
   complex plane, so that ln is not well defined outside the real line.
   There are ways around these difficulties (defining branches of the
   logarithm, for example), but we shall not go into this here.
   
   The operation of exponentiation has also been extended to other
   systems like matrices and operators. The key is to define an
   exponential function by (6) and work from there. [Some reference on
   operator calculus and/or advanced linear algebra?]
   
   
   
   References
   
   Complex Analysis. Ahlfors, Lars V. McGraw-Hill, 1953.
   
   
   
   
     _________________________________________________________________
   
   
   
    alopez-o@barrow.uwaterloo.ca
    Sun Nov 20 20:45:48 EST 1994


Archive-Name: sci-math-faq/fields
Last-modified: December 8, 1994
Version: 6.1



   
				FIELDS MEDAL
				       
   
   
   
     _________________________________________________________________
   
     * Historical Introduction
     * Table of Awardees
       
   
     _________________________________________________________________
   


   
Historical Introduction

   
   
   This is the original letter by Fields creating the endowment for the
   medals that bear his name. It is thought to have been written during
   the few onths before his death. Notice that no mention is made about
   the age of the recipients (currently there is a 40 year-old limit),
   and that the medal should not be attached to any person, private or
   public, meaning that it shouldn't bear anybody's name.
   
     It is proposed to found two gold medals to be awarded at successive
     International Mathematical Congress for outstanding achievements in
     mathematics. Because of the multiplicity of the branches of
     mathematics and taking into account the fact that the interval
     between such congresses is four years it is felt that at least two
     medals should be available. The awards would be open to the whole
     world and would be made by an International Comitee.
     
     The fund for the founding of the medals is constituted by balance
     left over after financing the Toronto congress held in 1924. This
     must be held in trust by the Government or by some body authorized
     by government to hold and invest such funds. It would seem that a
     dignified method for handling the matter and one which in this
     changing world should most nearly secure permanency would be for the
     Canadian Government to take over the fund and appoint as his
     custodian say the Prime Minister of the Dominion or the Prime
     Minister in association with the Minister of Finance. The medals
     would be struck at the Mint in Ottawa ans the duty of the custodian
     would be simply to hand over the medals at the proper time to the
     accredited International Committee.
     
     As things are at present a practical course of procedure would seem
     to be for the Executive Committee of a Congress to appoint a small
     international committee authorized to add to its number and call
     into consultation other mathematicians as it might demm expedient.
     The Committee would be expected to decide on the ones to whom the
     awards should be made thirty months in advance of the following
     Congress. Its decisions would be communicated to the President and
     Secretary of the Organizing Committee of the Congress, this
     Committee having the duty of communicating to the Prime Minister of
     Canada the names of the recipients in order that the medal might be
     prepared in time and forwarded to the president of the Organizing
     Committee. Immediately on the appointment of the Executive Committee
     of the Congress the medals would be handed over to its President.
     The presentation of the medals would constitute a special feature at
     some general meeting of the Congress.
     
     In the above arrangements the role of the Organizing Committee might
     be taken over by the Executive of the International Mathematical
     Union at some time in the future when that organization has benn
     generally accepted.
     
     In coming to its decision the hands of the IC should be left as free
     as possible. It would be understood, however, that in making the
     awards while it was in recognition of work already done it was at
     the same time intended to be an encouragement for further
     achievement on the part of the recipients and a stimulus to renewed
     effort on the part of others.
     
     In commenting on the work of the medalists it might be well to be
     conservative in one's statements to avoid envidious comparisons
     explicit or implied. The Committee might ease matters by saying they
     have decided to make the awards along certain lines not alone
     because of the outstanding character of the achievement but also
     with a view to encouraging further development along these lines. In
     this connection the Committe might say that they had elected to
     select subjects in Analysis, in Geometry, in the Theory of Groups,
     in the Theory of Numbers etc. as the case might be. When the
     Committee had come to an agreement in this sense the claims for
     recognition of work done along the special lines in question could
     be considered in detail by two smaller groups or subcommittees with
     specialized qualifications who would have authority to take into
     consultation or add to the subcommittees other mathematicians of
     specialized knowledge.
     
     With regard to the medals themselves, I might say that they should
     each contain at least 200 dollars worth of gold and be of a fair
     size, probably 7.5 centimeters in diameter. Because of the
     international character the language to be employed it would seem
     should be Latin or Greek? The design has still to be definitely
     determined. It will have to be decided on by artists in consultation
     with mathematicians. The suggestions made in the preceding are
     tentative and open to consideration on the part of mathematicians.
     
     It is not contemplated to make an award until 1936 at the Congress
     following that at Zurich during which an international Medal
     Committee should be named.
     
     The above programme means a new departure in the matter of
     international scientific cooperation and is likely to be the
     precursor of moves along like lines in other sciences than
     mathematics.
     
     One would hear again emphasized the fact that the medals should be
     of a character as purely international and impersonal as possible.
     There should not be attached to them in any way the name of any
     country, institution or person.
     
     Perhaps provision could be made as soon as possible after the
     appointment of the Executive of the Zurich Congress for the
     consideration by it of the subject of the medals, and the
     appointment without undue delay of a Commitee and the awards of the
     medals to be made in connection with the Congress of 1936.
     
     Suggestions with regard to the design of the medals will be welcome.
     
     
     (signed) J.C. Fields Research Professor of Mathematics University of
     Toronto
     
   
   
   
     _________________________________________________________________
   
   
   
Table of Awardees

   
   

Table of Awardees


Year Name                       Birthplace      Country         Age

1936 Ahlfors, Lars              Helsinki        Finland         29
1936 Douglas, Jesse             New York, NY    USA             39
1950 Schwartz, Laurent          Paris           France          35
1950 Selberg, Atle              Langesund       Norway          33
1954 Kodaira, Kunihiko          Tokyo           Japan           39
1954 Serre, Jean-Pierre         Bages           France          27
1958 Roth, Klaus                Breslau         Germany         32
1958 Thom, Rene                 Montbeliard     France          35
1962 Hormander, Lars            Mjallby         Sweden          31
1962 Milnor, John               Orange, NJ      USA             31
1966 Atiyah, Michael            London          UK              37
1966 Cohen, Paul                Long Branch NJ  USA             32
1966 Grothendieck, Alexander    Berlin          Germany         38
1966 Smale, Stephen             Flint, MI       USA             36
1970 Baker, Alan                London          UK              31
1970 Hironaka, Heisuke          Yamaguchi-ken   Japan           39
1970 Novikov, Serge             Gorki           USSR            32
1970 Thompson, John             Ottawa, KA      USA             37
1974 Bombieri, Enrico           Milan           Italy           33
1974 Mumford, David             Worth, Sussex   UK              37
1978 Deligne, Pierre            Brussels        Belgium         33
1978 Fefferman, Charles         Washington DC   USA             29
1978 Margulis, Gregori          Moscow          USSR            32
1978 Quillen, Daniel            Orange, NJ      USA             38
1982 Connes, Alain              Draguignan      France          35
1982 Thurston, William          Washington DC   USA             35
1982 Yau, Shing-Tung            Kwuntung        China           33
1986 Donaldson, Simon           Cambridge       UK              27
1986 Faltings, Gerd                             Germany         32
1986 Freedman, Michael          Los Angeles     USA             35
1990 Drinfeld, Vladimir         Kharkov         USSR            36
1990 Jones, Vaughan             Gisborne        N Zealand       38
1990 Mori, Shigefumi            Nagoya          Japan           39
1990 Witten, Edward             Baltimore       USA             38



Year Name                       Institution                             Country

1936 Ahlfors, Lars              Harvard University                      USA
1936 Douglas, Jesse             MIT                                     USA
1950 Schwartz, Laurent          Universite de Nancy                     France
1950 Selberg, Atle              Princeton/Inst. of Advanced Studies     USA
1954 Kodaira, Kunihiko          Princeton University                    USA
1954 Serre, Jean-Pierre         College de France                       France
1958 Roth, Klaus                University of London                    UK
1958 Thom, Rene                 University of Strasbourg                France
1962 Hormander, Lars            University of Stockholm                 Sweden
1962 Milnor, John               Princeton University                    USA
1966 Atiyah, Michael            Oxford University                       UK
1966 Cohen, Paul                Stanford University                     USA
1966 Grothendieck, Alex         University of Paris                     France
1966 Smale, Stephen             University of California at Berkeley    USA
1970 Baker, Alan                Cambridge University                    UK
1970 Hironaka, Heisuke          Harvard University                      USA
1970 Novikov, Serge             Moscow University                       USSR
1970 Thompson, John             University of Chicago                   USA
1974 Bombieri, Enrico           Univeristy of Pisa                      Italy
1974 Mumford, David             Harvard University                      USA
1978 Deligne, Pierre            IHES                                    France
1978 Fefferman, Charles         Princeton University                    USA
1978 Margulis, Gregori          InstPrblmInfTrans                       USSR
1978 Quillen, Daniel            MIT                                     USA
1982 Connes, Alain              IHES                                    France
1982 Thurston, William          Princeton University                    USA
1982 Yau, Shing-Tung            IAS                                     USA
1986 Donaldson, Simon           Oxford University                       UK
1986 Faltings, Gerd             Princeton University                    USA
1986 Freedman, Michael          University of California at San Diego   USA
1990 Drinfeld, Vladimir         Phys.Inst.Kharkov                       USSR
1990 Jones, Vaughan             University of California at Berkeley    USA
1990 Mori, Shigefumi            University of Kyoto?                    Japan
1990 Witten, Edward             Princeton/Institute of Advanced Studies USA


   
   
   References
   
   International Mathematical Congresses, An Illustrated History
   1893-1986. Donald J.Alberts, G. L. Alexanderson and Constance Reid.
   Revised Edition, Including 1986, Springer Verlag, 1987.
   
   
   
   Tropp, Henry S. The origins and history of the Fields Medal. Historia
   Mathematica, 3(1976), 167-181.
   
   
   
   
   
   
     _________________________________________________________________
   
   
   
    alopez-o@barrow.uwaterloo.ca
    Sun Nov 20 20:45:48 EST 1994


Archive-Name: sci-math-faq/FLT/Fermat
Last-modified: December 8, 1994
Version: 6.1


   
   
   
Did Fermat prove this theorem?

   
   
   No he did not. Fermat claimed to have found a proof of the theorem at
   an early stage in his career. Much later he spent time and effort
   proving the cases n = 4 and n = 5 . Had he had a proof to his theorem,
   there would have been no need for him to study specific cases.
   
   Fermat may have had one of the following ``proofs'' in mind when he
   wrote his famous comment.
   
     * Fermat discovered and applied the method of infinite descent,
       which, in particular can be used to prove FLT for n = 4 . This
       method can actually be used to prove a stronger statement than FLT
       for n = 4 , viz, x^4 + y^4 = z^2 has no non-trivial integer
       solutions. It is possible and even likely that he had an incorrect
       proof of FLT using this method when he wrote the famous
       ``theorem''.
     * He had a wrong proof in mind. The following proof, proposed first
       by Lame' was thought to be correct, until Liouville pointed out
       the flaw, and by Kummer which latter became and expert in the
       field. It is based on the incorrect assumption that prime
       decomposition is unique in all domains.
       
       The incorrect proof goes something like this:
       
       We only need to consider prime exponents (this is true). So
       consider x^p + y^p = z^p . Let r be a primitive p -th root of
       unity (complex number)
       
       Then the equation is the same as:
       
       (x + y)(x + ry)(x + r^2y)...(x + r^(p - 1)y) = z^p
       
       Now consider the ring of the form:
       
       a_1 + a_2 r + a_3 r^2 + ... + a_(p - 1) r^(p - 1)
       
       where each a_i is an integer
       
       Now if this ring is a unique factorization ring (UFR), then it is
       true that each of the above factors is relatively prime. From this
       it can be proven that each factor is a pth power and from this FLT
       follows.
       
       The problem is that the above ring is not an UFR in general.
       
   
   
   Another argument for the belief that Fermat had no proof -and,
   furthermore, that he knew that he had no proof- is that the only place
   he ever mentioned the result was in that marginal comment in Bachet's
   Diophantus. If he really thought he had a proof, he would have
   announced the result publicly, or challenged some English
   mathematician to prove it. It is likely that he found the flaw in his
   own proof before he had a chance to announce the result, and never
   bothered to erase the marginal comment because it never occurred to
   him that anyone would see it there.
   
   Some other famous mathematicians have speculated on this question.
   Andre Weil, writes:
   
     Only on one ill-fated occasion did Fermat ever mention a curve of
     higher genus x^n + y^n = z^n , and then hardly remain any doubt that
     this was due to some misapprehension on his part [for a brief moment
     perhaps [he must have deluded himself into thinking he had the
     principle of a general proof.
     
   
   
   Winfried Scharlau and Hans Opolka report:
   
     Whether Fermat knew a proof or not has been the subject of many
     speculations. The truth seems obvious ...[Fermat's marginal note]
     was made at the time of his first letters concerning number theory
     [1637]...as far as we know he never repeated his general remark, but
     repeatedly made the statement for the cases n = 3 and 4 and posed
     these cases as problems to his correspondents [he formulated the
     case n = 3 in a letter to Carcavi in 1659 [All these facts indicate
     that Fermat quickly became aware of the incompleteness of the
     [general] ``proof" of 1637. Of course, there was no reason for a
     public retraction of his privately made conjecture.
     
   
   
   However it is important to keep in mind that Fermat's ``proof"
   predates the Publish or Perish period of scientific research in which
   we are still living.
   
   
   
   References
   
   From Fermat to Minkowski: lectures on the theory of numbers and its
   historical development. Winfried Scharlau, Hans Opolka. New York,
   Springer, 1985.
   
   
   
   Basic Number Theory. Andre Weil. Berlin, Springer, 1967
   
   
   
   
   
   
     _________________________________________________________________
   
   
   
    alopez-o@barrow.uwaterloo.ca
    Sun Nov 20 20:45:48 EST 1994


Archive-Name: sci-math-faq/FLT/status 
Last-modified: December 8, 1994
Version: 6.1



What is the current status of FLT?


   
   Theorem 1 (Fermat's Last Theorem) There are no positive integers x, y,
   z, and n>2 such that x^n + y^n = z^n.
   
   Andrew Wiles, a researcher at Princeton, claims to have found a proof.
   The proof was presented in Cambridge, UK during a three day seminar to
   an audience which included some of the leading experts in the field.
   The proof is long and cumbersome. At this point the reviewers have
   found a glitch in the argument. Late last year, Prof. Wiles
   acknowledged that a gap existed. On October 25th, 1994. Prof. Andrew
   Wiles released two preprints, Modular elliptic curves and Fermat's
   Last Theorem, by Andrew Wiles, and Ring theoretic properties of
   certain Hecke algebras, by Richard Taylor and Andrew Wiles.
   
   The first one (long) announces a proof of, among other things,
   Fermat's Last Theorem, relying on the second one (short) for one
   crucial step.
   
   The argument described by Wiles in his Cambridge lectures had a
   serious gap, namely the construction of an Euler system. After trying
   unsuccessfully to repair that construction, Wiles went back to a
   different approach he had tried earlier but abandoned in favor of the
   Euler system idea. He was able to complete his proof, under the
   hypothesis that certain Hecke algebras are local complete
   intersections. This and the rest of the ideas described in Wiles'
   Cambridge lectures are written up in the first manuscript. Jointly,
   Taylor and Wiles establish the necessary property of the Hecke
   algebras in the second paper.
   
   The new approach turns out to be significantly simpler and shorter
   than the original one, because of the removal of the Euler system. (In
   fact, after seeing these manuscripts Faltings has apparently come up
   with a further significant simplification of that part of the
   argument.)
   
   The preprints were submitted to The Annals of Mathematics. According
   to the New York Times the new proof has been vetted by four
   researchers already, who have found no mistake.
   
   
     _________________________________________________________________
   
   
   
    alopez-o@barrow.uwaterloo.ca
    Sun Nov 20 20:45:48 EST 1994


Archive-Name: sci-math-faq/FLT/Wrong
Last-modified: December 8, 1994
Version: 6.1


  
If not, then what?

   
   FLT is usually broken into 2 cases. The first case assumes (abc,n) = 1
   . The second case is the general case.
   
  WHAT HAS BEEN PROVED
  
   
   
   First Case.
   
   It has been proven true up to 7.568x10^(17) by the work of Wagstaff
   &Tanner, Granville &Monagan, and Coppersmith. They all used extensions
   of the Wiefrich criteria and improved upon work performed by Gunderson
   and Shanks &Williams.
   
   The first case has been proven to be true for an infinite number of
   exponents by Adelman, Frey, et. al. using a generalization of the
   Sophie Germain criterion
   
   Second Case:
   
   It has been proven true up to n = 150,000 by Tanner &Wagstaff. The
   work used new techniques for computing Bernoulli numbers mod p and
   improved upon work of Vandiver. The work involved computing the
   irregular primes up to 150,000. FLT is true for all regular primes by
   a theorem of Kummer. In the case of irregular primes, some additional
   computations are needed. More recently, Fermat's Last Theorem has been
   proved true up to exponent 4,000,000 in the general case. The method
   used was essentially that of Wagstaff: enumerating and eliminating
   irregular primes by Bernoulli number computations. The computations
   were performed on a set of NeXT computers by Richard Crandall et al.
   
   Since the genus of the curve a^n + b^n = 1 , is greater than or equal
   to 2 for n > 3 , it follows from Mordell's theorem [proved by
   Faltings], that for any given n , there are at most a finite number of
   solutions.
   
  CONJECTURES
  
   
   
   There are many open conjectures that imply FLT. These conjectures come
   from different directions, but can be basically broken into several
   classes: (and there are interrelationships between the classes)
    1. Conjectures arising from Diophantine approximation theory such as
       the ABC conjecture, the Szpiro conjecture, the Hall conjecture,
       etc.
       
       For an excellent survey article on these subjects see the article
       by Serge Lang in the Bulletin of the AMS, July 1990 entitled ``Old
       and new conjectured diophantine inequalities".
       
       Masser and Osterle formulated the following known as the ABC
       conjecture:
       
       Given epsilon > 0 , there exists a number C(epsilon) such that for
       any set of non-zero, relatively prime integers a,b,c such that a +
       b = c we have max (|a|, |b|, |c|) <= C(epsilon) N(abc)^(1 +
       epsilon) where N(x) is the product of the distinct primes dividing
       x .
       
       It is easy to see that it implies FLT asymptotically. The
       conjecture was motivated by a theorem, due to Mason that
       essentially says the ABC conjecture is true for polynomials.
       
       The ABC conjecture also implies Szpiro's conjecture [and
       vice-versa] and Hall's conjecture. These results are all generally
       believed to be true.
       
       There is a generalization of the ABC conjecture [by Vojta] which
       is too technical to discuss but involves heights of points on
       non-singular algebraic varieties . Vojta's conjecture also implies
       Mordell's theorem [already known to be true]. There are also a
       number of inter-twined conjectures involving heights on elliptic
       curves that are related to much of this stuff. For a more complete
       discussion, see Lang's article.
       
    2. Conjectures arising from the study of elliptic curves and modular
       forms. - The Taniyama-Weil-Shmimura conjecture.
       
       There is a very important and well known conjecture known as the
       Taniyama-Weil-Shimura conjecture that concerns elliptic curves.
       This conjecture has been shown by the work of Frey, Serre, Ribet,
       et. al. to imply FLT uniformly, not just asymptotically as with
       the ABC conj.
       
       The conjecture basically states that all elliptic curves can be
       parameterized in terms of modular forms.
       
       There is new work on the arithmetic of elliptic curves. Sha, the
       Tate-Shafarevich group on elliptic curves of rank 0 or 1. By the
       way an interesting aspect of this work is that there is a close
       connection between Sha, and some of the classical work on FLT. For
       example, there is a classical proof that uses infinite descent to
       prove FLT for n = 4 . It can be shown that there is an elliptic
       curve associated with FLT and that for n = 4 , Sha is trivial. It
       can also be shown that in the cases where Sha is non-trivial, that
       infinite-descent arguments do not work; that in some sense 'Sha
       blocks the descent'. Somewhat more technically, Sha is an
       obstruction to the local-global principle [e.g. the
       Hasse-Minkowski theorem].
       
    3. Conjectures arising from some conjectured inequalities involving
       Chern classes and some other deep results/conjectures in
       arithmetic algebraic geometry.
       
       This results are quite deep. Contact Barry Mazur [or Serre, or
       Faltings, or Ribet, or ...]. Actually the set of people who DO
       understand this stuff is fairly small.
       
       The diophantine and elliptic curve conjectures all involve deep
       properties of integers. Until these conjecture were tied to FLT,
       FLT had been regarded by most mathematicians as an isolated
       problem; a curiosity. Now it can be seen that it follows from some
       deep and fundamental properties of the integers. [not yet proven
       but generally believed].
       
   
   
   This synopsis is quite brief. A full survey would run to many pages.
   
   
   
   References
   
   [1] J.P.Butler, R.E.Crandall,&R.W.Sompolski, Irregular Primes to One
   Million. Math. Comp., 59 (October 1992) pp. 717-722.
   
   
   
   Fermat's Last Theorem, A Genetic Introduction to Algebraic Number
   Theory. H.M. Edwards. Springer Verlag, New York, 1977.
   
   
   
   Thirteen Lectures on Fermat's Last Theorem. P. Ribenboim. Springer
   Verlag, New York, 1979.
   
   
   
   Number Theory Related to Fermat's Last Theorem. Neal Koblitz, editor.
   Birkhduser Boston, Inc., 1982, ISBN 3-7643-3104-6
   
   
   
   
     _________________________________________________________________
   
   
   
    alopez-o@barrow.uwaterloo.ca
    Sun Nov 20 20:45:48 EST 1994


Archive-Name: sci-math-faq/FLT/history 
Last-modified: December 8, 1994
Version: 6.1

   
   
   
History of Fermat's Last Theorem

   
   
   Pierre de Fermat (1601-1665) was a lawyer and amateur mathematician.
   In about 1637, he annotated his copy (now lost) of Bachet's
   translation of Diophantus' Arithmetika with the following statement:
   
     Cubem autem in duos cubos, aut quadratoquadratum in duos
     quadratoquadratos, et generaliter nullam in infinitum ultra
     quadratum potestatem in duos ejusdem nominis fas est dividere:
     cujus rei demonstrationem mirabilem sane detexi. Hanc marginis
     exiguitas non caparet.
     
   In English, and using modern terminology, the paragraph above reads
   as:
   
     There are no positive integers such that x^n + y^n = z^n for n > 2 .
     I've found a remarkable proof of this fact, but there is not enough
     space in the margin [of the book] to write it.
     
   Fermat never published a proof of this statement. It became to be
   known as Fermat's Last Theorem (FLT) not because it was his last piece
   of work, but because it is the last remaining statement in the
   post-humous list of Fermat's works that needed to be proven or
   independently verified. All others have either been shown to be true
   or disproven long ago.
   
   
     _________________________________________________________________
   
   
   
    alopez-o@barrow.uwaterloo.ca
    Sun Nov 20 20:45:48 EST 1994


Archive-Name: sci-math-faq/specialnumbers/fxtofxeqx
Last-modified: December 8, 1994
Version: 6.1



Name for f(x)^(f(x)) = x 

   
   Solving for f one finds a "continued fraction"-like answer

f(x)=log(x)/(log(log(x)/log(log(x)/(log...)))

   This question has been repeated here from time to time over the years,
   and no one seems to have heard of any published work on it, nor a
   published name for it (D. Merrit proposes ``lx" due to its (very)
   faint resemblance to log). It's not an analytic function.
   
   The ``continued fraction" form for its numeric solution is highly
   unstable in the region of its minimum at 1/e (because the graph is
   quite flat there yet logarithmic approximation oscillates wildly),
   although it converges fairly quickly elsewhere. To compute its value
   near 1/e , use the bisection method which gives good results.
   Bisection in other regions converges much more slowly than the
   logarithmic continued fraction form, so a hybrid of the two seems
   suitable. Note that it's dual valued for the reals (and many valued
   complex for negative reals).
   
   A similar function is a built-in function in MAPLE called W(x) or
   Lambert's W function. MAPLE considers a solution in terms of W(x) as a
   closed form (like the erf function). W is defined as W(x)e^(W(x)) = x
   .
   
   Notice that f(x) = exp(W(log(x))) is the solution to f(x)^f(x) = x
   
   An extensive treatise on the known facts of Lambert's W function is
   available for anonymous ftp at dragon.uwaterloo.ca at
   /cs-archive/CS-93-03/W.ps.Z.
   
   
   
   
     _________________________________________________________________
   
   
   
    alopez-o@barrow.uwaterloo.ca
    Sun Nov 20 20:45:48 EST 1994

Archive-Name: sci-math-faq/hilbert 
Last-modified: December 8, 1994
Version: 6.1

   
		     WHICH ARE THE 23 HILBERT PROBLEMS?
				       
   
   
   The original was published in German in a couple of places. A
   translation was published by the AMS in 1902.
   
   
   
   D. Hilbert. Mathematical problems. Lecture delivered before the
   International Congress of Mathematicians at Paris in 1900. Bulletin
   of the American Mathematical Society, 8:437-479, 1902.
   
   
   
   The AMS Symposium mentioned at the end contains a series of papers on
   the then-current state of most of the Problems.
   
   Incidentally, the bibliography information above is taken from Yuri
   Matiyasevich's book "Hilbert's Tenth Problem", published last year by
   MIT Press. The full bibliography is available via anonymous ftp from
   theory.lcs.mit.edu in the directory pub/hilbert10 (which also contains
   some other information about the book).
   
   
   
   
     _________________________________________________________________
   
   
   
    alopez-o@barrow.uwaterloo.ca
    Sun Nov 20 20:45:48 EST 1994



Archive-Name: sci-math-faq/montyhall
Last-modified: December 8, 1994
Version: 6.1


The Monty Hall problem

   
   
   This problem has rapidly become part of the mathematical folklore.
   
   The American Mathematical Monthly, in its issue of January 1992,
   explains this problem carefully. The following are excerpted from that
   article.
   
   Problem:
   
   A TV host shows you three numbered doors (all three equally likely),
   one hiding a car and the other two hiding goats. You get to pick a
   door, winning whatever is behind it. Regardless of the door you
   choose, the host, who knows where the car is, then opens one of the
   other two doors to reveal a goat, and invites you to switch your
   choice if you so wish. Does switching increases your chances of
   winning the car?
   
   If the host always opens one of the two other doors, you should
   switch. Notice that 1/3 of the time you choose the right door (i.e.
   the one with the car) and switching is wrong, while 2/3 of the time
   you choose the wrong door and switching gets you the car.
   
   Thus the expected return of switching is 2/3 which improves over your
   original expected gain of 1/3 .
   
   Even if the hosts switches only part of the time, it pays to switch.
   Only in the case where we assume a malicious host (i.e. a host who
   entices you to switch based in the knowledge that you have the right
   door) would it pay not to switch.
   
   
   
   References
   
   L. Gillman The Car and the Goats American Mathematical Monthly,
   January 1992, pp. 3-7.
   
   
   
   
     _________________________________________________________________
   
   
   
    alopez-o@barrow.uwaterloo.ca
    Sun Nov 20 20:45:48 EST 1994


Archive-Name: sci-math-faq/mastermind
Last-modified: December 8, 1994
Version: 6.1


   
   
Master Mind

   
   
   For the game of Master Mind it has been proven that no more than five
   moves are required in the worst case.
   
   One such algorithm was published in the Journal of Recreational
   Mathematics; in '70 or '71 (I think), which always solved the 4 peg
   problem in 5 moves. Knuth later published an algorithm which solves
   the problem in a shorter number of moves - on average - but can take
   six guesses on certain combinations.
   
   In 1994, Kenji Koyama and Tony W. Lai found, by exhaustive search that
   5625/1296~= 4.340 is the optimal strategy in the expected case. This
   strategy may take six guesses in the worst case. A strategy that uses
   at most five guesses in the worst case is also shown. This strategy
   requires 5626/1296~= 4.341 guesses.
   
   
   
   References
   
   Donald E. Knuth. The Computer as Master Mind. J. Recreational
   Mathematics, 9 (1976-77), 1-6.
   
   
   
   Kenji Koyama, Tony W. Lai. An optimal Mastermind Strategy. J.
   Recreational Mathematics, 1994.
   
   
   
   
   
   
     _________________________________________________________________
   
   
   
    alopez-o@barrow.uwaterloo.ca
    Sun Nov 20 20:45:48 EST 1994


Archive-Name: sci-math-faq/numbers
Last-modified: December 8, 1994
Version: 6.1


   
   

			      WHAT ARE NUMBERS?




     _________________________________________________________________

     * Introduction
     * Construction of the Number System
     * Construction of N
     * Construction of Z
     * Construction of Q
     * Construction of R
     * Construction of C
     * Rounding things up
     * What's next?


     _________________________________________________________________



Introduction

   
   
   Informally:
     * \N = { 0,1,... } or \N = { 1,2,... }
       Wether 0 is in \N depends on where you live and what is your field
       of interest. At the informal level it is a religious topic.
     * \Z = { ..., - 1,0,1,... }
     * \Q = { p/q | p, q in \Z and q != 0 }
     * \R = { d_0.d_1d_2... | d_0 in \Z and 0 <= d_i <= 9 for i > 0 }
     * \C = { a + b o i | a, b in \R and i^2 = -1 }
       
   
   
   
     _________________________________________________________________
   
   
   
Construction of the Number System

   
   
   Formally (following the main stream in math) the numbers are
   constructed from scratch out of the axioms of Zermelo Fraenkel set
   theory (a.k.a. ZF set theory) [Enderton77, Henle86, Hrbacek84]. The
   only things that can be derived from the axioms are sets with the
   empty set at the bottom of the hierarchy. This will mean that any
   number is a set (it is the only thing you can derive from the axioms).
   It doesn't mean that you always have to use set notation when you use
   numbers: just introduce the ``informal'' numbers as an abbreviation of
   the formal counterparts.
   
   The construction starts with \N and algebraically speaking, \N with
   its operations and order is quite a weak structure. In the following
   constructions the structures will be strengthen one step at the time:
   \Z will be an integral domain, \Q will be a field, for the field \R
   the order will be made complete, and field \C will be made
   algebraically complete.
   
   Before we start, first some notational stuff:
     * a pair (a,b) = { { a } , { a,b } } ,
     * an equivalence class [a] = { b | a == b } ,
     * the successor of a is s(a) = a U { a } .
       
   
   
   
     _________________________________________________________________
   
   
   
Construction of N

   
   
     * { } in \N
     * if a in \N then s(a) in \N
     * \N is the smallest possible set such that the preceding rules
       hold.
       
   Informally n = { 0,...,n - 1 } (thus 0 = { } , 1 = { 0 } , 2 = { 0,1 }
   , 3 = { 0,1,2 } ). We will refer to the elements of \N by giving them
   a subscript _n . The relation <_n on \N is defined as: a_n <_n b_n iff
   a_n in b_n . We can define +_n as follows:
     * a_n +_n 0_n = a_n
     * a_n +_n s(b_n) = s(a_n +_n b_n)
       
   Define *_n as:
     * a_n *_n 0_n = 0_n
     * a_n *_n s(b_n) = (a_n *_n b_n) +_n a_n
       
   
   
   
     _________________________________________________________________
   
   
Construction of Z

   
   
   We define an equivalence relation on \N x \N : (a_n,b_n) ==_z(c_n,d_n)
   iff a_n +_n d_n = c_n +_n b_n . Note that ==_z ``simulates'' a
   subtraction in \N . \Z = { [(a_n,b_n)]_z | a_n, b_n in \N } . We will
   refer to the elements of \Z by giving them a subscript _z . The
   elements of \N can be embedded as follows: embed_n : \N --> \Z such
   that embed_n(a_n) = [(a_n,0_n)]_z . Furthermore we can define:
     * [(a_n,b_n)]_z <_z [(c_n,d_n)]_z iff a_n +_n d_n <_n c_n +_n b_n
     * [(a_n,b_n)]_z +_z [(c_n,d_n)]_z = [(a_n +_n c_n, b_n +_n d_n)]_z
     * [(a_n,b_n)]_z *_z [(c_n,d_n)]_z =
       [((a_n *_n c_n) +_n (b_n *_n d_n), (a_n *_n d_n) +_n (c_n *_n
       b_n))]_z
       
   
   
   
     _________________________________________________________________
   
   
Construction of Q

   
   
   We define an equivalence relation on \Z x (\Z \ { 0_z }) : (a_z,b_z)
   ==_q (c_z,d_z) iff a_z *_z d_z = c_z *_z b_z . Note that ==_q
   ``simulates'' a division in \Z . \Q = { [(a_z,b_z)]_q | a_z in \Z and
   b_z in \Z \ { 0_z } } . We will refer to the elements of \Q by giving
   them a subscript _q . The elements of \Z can be embedded as follows:
   embed_z : \Z --> \Q such that embed_z(a_z) = [(a_z,1_z)]_q .
   Furthermore we can define:
     * [(a_z,b_z)]_q <_q [(c_z,d_z)]_q iff a_z *_z d_z <_z c_z *_z b_z
       when 0_z <_z b_z and 0_z <_z d_z
     * [(a_z,b_z)]_q +_q [(c_z,d_z)]_q = [((a_z *_z d_z) +_z (c_z *_z
       b_z), b_z *_z d_z)]_q
     * [(a_z,b_z)]_q *_q [(c_z,d_z)]_q = [(a_z *_z c_z, b_z *_z d_z)]_q
       
   
   
   
     _________________________________________________________________
   
   
   
Construction of R

   
   
   The construction of \R is different (and more awkward to understand)
   because we must ensure that the cardinality of \R is greater than that
   of \Q .
   Set c is a Dedekind cut iff
     * { } subset c subset \Q (strict inclusions!)
     * c is closed downward:
       if a_q in c and b_q <_q a_q then b_q in c
     * c has no largest element:
       there isn't an element a_q in c such that b_q <_q a_q for all b_q
       != a_q in c
       
   You can think of a cut as taking a pair of scissors and cutting \Q in
   two parts such that one part contains all the small numbers and the
   other part contains all large numbers. If the part with the small
   numbers was cut in such a way that it doesn't have a largest element,
   it is called a Dedekind cut. \R = { c | c is a Dedekind cut } . We
   will refer to the elements of \R by giving them a subscript _r . The
   elements of \Q can be embedded as follows: embed_q : \Q --> \R such
   that embed_q(a_q) = { b_q | b_q <_q a_q } . Furthermore we can define:
     * a_r <_r b_r iff a_r subset b_r (strict inclusion!)
     * a_r +_r b_r = { c_q +_q d_q | c_q in a_r and d_q in b_r }
     * -_r a_r = ; { b_q | there exists an c_q in \Q such that b_q <_q
       c_q and (-1)_q *_q c_q \not in a_r }
     * |a_r|_r = a_r U -_r a_r
     * *_r is defined as:
	  + if not a_r <_r 0_r and not b_r <_r 0_r
	    then a_r *_r b_r = 0_r U { c_q *_q d_q | c_q in a_r and d_q
	    in b_r }
	  + if a_r <_r 0_r and b_r <_r 0_r then a_r *_r b_r = |a_r|_r *_r
	    |b_r|_r
	  + otherwise a_r *_r b_r = -_r (|a_r|_r *_r |b_r|_r)
	    
   
   
   There exists an alternative definition of \R using Cauchy sequences: a
   Cauchy sequence is a s : \N --> \Q such that s(i_n) +_q((-1)_q *_q
   s(j_n)) can be made arbitrary near to 0_q for all sufficiently large
   i_n and j_n . We will define an equivalence relation ==_r on the set
   of Cauchy sequences as: r ==_r s iff r(m_n) +_q((-1)_q *_q s(m_n)) can
   be made arbitrary close to 0_q for all sufficiently large m_n . \R = {
   [s]_r | s is a Cauchy sequence } . Note that this definition is close
   to ``decimal'' expansions.
   
   
     _________________________________________________________________
   
   
   
Construction of C

   
   
   \C = \R x \R . We will refer to the elements of \C by giving them a
   subscript _c . The elements of \R can be embedded as follows: embed_r
   : \R --> \C such that embed_r(a_r) = (a_r,0_r) . Furthermore we can
   define:
     * (a_r,b_r) +_c (c_r,d_r) = (a_r +_r c_r, b_r +_r d_r )
     * (a_r,b_r) *_c (c_r,d_r) = ((a_r *_r c_r) +_r -_r (b_r * d_r), (a_r
       *_r d_r) +_r (b_r *_r c_r))
       
   
   
   There exists an elegant alternative definition using ideals. To be a
   bit sloppy: \C = \R[x]/< (x *_r x) +_r 1_r > , i.e. \C is the
   resulting quotient ring of factoring ideal < (x *_r x) +_r 1_r > out
   of the ring \R[x] of polynomials over \R . The sloppy part is that we
   need to define concepts like quotient ring, ideal, and ring of
   polynomials. Note that this definition is close to working with i^2 =
   -1 : (x *_r x) +_r 1_r = 0_r can be rewritten as (x *_r x) = (-1)_r .
   
   
     _________________________________________________________________
   
   
   
Construction of C

   
   
   \C = \R x \R . We will refer to the elements of \C by giving them a
   subscript _c . The elements of \R can be embedded as follows: embed_r
   : \R --> \C such that embed_r(a_r) = (a_r,0_r) . Furthermore we can
   define:
     * (a_r,b_r) +_c (c_r,d_r) = (a_r +_r c_r, b_r +_r d_r )
     * (a_r,b_r) *_c (c_r,d_r) = ((a_r *_r c_r) +_r -_r (b_r * d_r), (a_r
       *_r d_r) +_r (b_r *_r c_r))
       
   
   
   There exists an elegant alternative definition using ideals. To be a
   bit sloppy: \C = \R[x]/< (x *_r x) +_r 1_r > , i.e. \C is the
   resulting quotient ring of factoring ideal < (x *_r x) +_r 1_r > out
   of the ring \R[x] of polynomials over \R . The sloppy part is that we
   need to define concepts like quotient ring, ideal, and ring of
   polynomials. Note that this definition is close to working with i^2 =
   -1 : (x *_r x) +_r 1_r = 0_r can be rewritten as (x *_r x) = (-1)_r .
   
   
     _________________________________________________________________
   
   
   
Rounding things up

   
   
   At this moment we don't have that \N is a subset of \Z , \Z of \Q ,
   etc. But we can get the inclusions if we look at the embedded copies
   of \N , \Z , etc. Let
     * \N' = ran embed_r o embed_q o embed_z o embed_n
     * \Z' = ran embed_r o embed_q o embed_z
     * \Q' = ran embed_r o embed_q
     * \R' = ran embed_r
       
   For these sets we have \N' subseteq \Z' subseteq \Q' subseteq \R'
   subseteq \C . Furthermore these sets have all the properties that the
   ``informal'' numbers have.
   
   
     _________________________________________________________________
   
  
This section of the FAQ is (C) Hans de Vreught. Send comments and or
corrections relating to this part to:

hdev@cp.tn.tudelft.nl (Hans de Vreught)



 
   


Archive-Name: sci-math-faq/nobel
Last-modified: December 8, 1994
Version: 6.1

   
   
     _________________________________________________________________
   
   Next: Who is N. Up: Mathematical Oddities Previous: Erdos Number
     _________________________________________________________________
   
   
   
Why is there no Nobel in mathematics? 

   
   
   Nobel prizes were created by the will of Alfred Nobel, a notable
   Swedish chemist.
   
   One of the most common -and unfounded- reasons as to why Nobel decided
   against a Nobel prize in math is that [a woman he proposed to/his
   wife/his mistress] [rejected him because of/cheated him with] a famous
   mathematician. Gosta Mittag-Leffler is often claimed to be the guilty
   party.
   
   There is no historical evidence to support the story.
   
   For one, Mr. Nobel was never married.
   
   There are more credible reasons as to why there is no Nobel prize in
   math. Chiefly among them is simply the fact he didn't care much for
   mathematics, and that it was not considered a practical science from
   which humanity could benefit (a chief purpose for creating the Nobel
   Foundation).
   
   Here are some relevant facts:
   
     * Nobel never married, hence no ``wife''. (He did have a mistress, a
       Viennese woman named Sophie Hess.)
     * Gosta Mittag-Leffler was an important mathematician in Sweden in
       the late 19th-early 20th century. He was the founder of the
       journal Acta Mathematica, played an important role in helping the
       career of Sonya Kovalevskaya, and was eventually head of the
       Stockholm Hogskola, the precursor to Stockholms Universitet.
       However, it seems highly unlikely that he would have been a
       leading candidate for an early Nobel Prize in mathematics, had
       there been one - there were guys like Poincare and Hilbert around,
       after all.
     * There is no evidence that Mittag-Leffler had much contact with
       Alfred Nobel (who resided in Paris during the latter part of his
       life), still less that there was animosity between them for
       whatever reason. To the contrary, towards the end of Nobel's life
       Mittag-Leffler was engaged in ``diplomatic'' negotiations to try
       to persuade Nobel to designate a substantial part of his fortune
       to the Hogskola. It seems hardly likely that he would have
       undertaken this if there was prior bad blood between them.
       Although initially Nobel seems to have intended to do this,
       eventually he came up with the Nobel Prize idea - much to the
       disappointment of the Hogskola, not to mention Nobel's relatives
       and Fraulein Hess.
     * According to the very interesting study by Elisabeth Crawford,
       ``The Beginnings of the Nobel Institution'', Cambridge Univ.
       Press, 1984, pages 52-53:
       
     Although it is not known how those in responsible positions at the
     Hogskola came to believe that a *large* bequest was forthcoming,
     this indeed was the expectation, and the disappointment was keen
     when it was announced early in 1897 that the Hogskola had been left
     out of Nobel's final will in 1895. Recriminations followed, with
     both Pettersson and Arrhenius [academic rivals of Mittag-Leffler in
     the administration of the Hogskola] letting it be known that Nobel's
     dislike for Mittag-Leffler had brought about what Pettersson termed
     the `Nobel Flop'. This is only of interest because it may have
     contributed to the myth that Nobel had planned to institute a prize
     in mathematics but had refrained because of his antipathy to
     Mittag-Leffler or -in another version of the same story- because of
     their rivalry for the affections of a woman....
     * A final speculation concerning the psychological element. Would
       Nobel, sitting down to draw up his testament, presumably in a mood
       of great benevolence to mankind, have allowed a mere personal
       grudge to distort his idealistic plans for the monument he would
       leave behind?
       
   
   
   Nobel, an inventor and industrialist, did not create a prize in
   mathematics simply because he was not particularly interested in
   mathematics or theoretical science. His will speaks of prizes for
   those ``inventions or discoveries'' of greatest practical benefit to
   mankind. (Probably as a result of this language, the physics prize has
   been awarded for experimental work much more often than for advances
   in theory.)
   
   However, the story of some rivalry over a woman is obviously much more
   amusing, and that's why it will probably continue to be repeated.
   
   
   
   References
   
   Mathematical Intelligencer, vol. 7 (3), 1985, p. 74.
   
   
   
   The Beginnings of the Nobel Institution. Elisabeth Crawford. Cambridge
   Univ. Press, 1984.
   
   
   
   
   
   
     _________________________________________________________________
   
   
   
    alopez-o@barrow.uwaterloo.ca
    Sun Nov 20 20:45:48 EST 1994


Archive-Name: sci-math-faq/specialnumbers/lawPieq3
Last-modified: December 8, 1994
Version: 6.1

   

Indiana bill sets the value of pi to 3

   
   
   The bill House Bill No. 246, Indiana State Legislature, 1897,
   reportedly set the value of pi to an incorrect rational approximation.
   
   
   The following is the text of the bill:
   
     HOUSE BILL NO. 246
     
     "A bill for an act introducing a new mathematical truth and offered
     as a contribution to education to be used only by the State of
     Indiana free of cost by paying any royalties whatever on the same,
     provided it is accepted and adopted by the official action of the
     legislature of 1897.
     
     "Section 1. Be it enacted by the General Assembly of the State of
     Indiana: It has been found that a circular area is to the square on
     a line equal to the quadrant of the circumference, as the area of an
     equilateral rectangle is to the square on one side. The diameter
     employed as the linear unit according to the present rule in
     computing the circle's area is entirely wrong, as it represents the
     circles area one and one-fifths times the area of a square whose
     perimeter is equal to the circumference of the circle. This is
     because one-fifth of the diameter fils to be represented four times
     in the circle's circumference. For example: if we multiply the
     perimeter of a square by one-fourth of any line one-fifth greater
     than one side, we can, in like manner make the square's area to
     appear one fifth greater than the fact, as is done by taking the
     diameter for the linear unit instead of the quadrant of the circle's
     circumference.
     
     "Section 2. It is impossible to compute the area of a circle on the
     diameter as the linear unit without tresspassing upon the area
     outside the circle to the extent of including one-fifth more area
     than is contained within the circle's circumference, because the
     square on the diameter produces the side of a square which equals
     nine when the arc of ninety degrees equals eight. By taking the
     quadrant of the circle's circumference for the linear unit, we
     fulfill the requirements of both quadrature and rectification of the
     circle's circumference. Furthermore, it has revealed the ratio of
     the chord and arc of ninety degrees, which is as seven to eight, and
     also the ratio of the diagonal and one side of a square which is as
     ten to seven, disclosing the fourth important fact, that the ratio
     of the diameter and circumference is as five-fourths to four; and
     because of these facts and the further fact that the rule in
     prresent use fails to work both ways mathematically, it should be
     discarded as wholly wanting and misleading in its practical
     applications.
     
     "Section 3. In further proof of the value of the author's proposed
     contribution to education, and offered as a gift to the State of
     Indiana, is the fact of his solutions of the trisection of the
     angle, duplication of the cube and quadrature having been already
     accepted as contributions to science by the American Mathematical
     Monthly, the leading exponent of mathematical thought in this
     country. And be it remembered that these noted problems had been
     long since given up by scientific bodies as unsolvable mysteries and
     above man's ability to comprehend."
     
   
   
   Will E. Edington in an article published in the Proceedings of the
   Indiana Academy of Science describes the fate of the bill in the
   committees of the Indiana legislature. First it was referred to the
   House Committee on Canals, which was also referred to as the Committee
   on Swamp Lands. Notices of the bill appeared in the Indianapolis
   Journal and the Indianapolis Sentinel on Jan. 19,1897, both of which
   described it a a bill telling how to square circles. On the same day,
   "Representative M.B.Butler, of Steuben County, chairman of the
   Committee on Canals, submitted the following report:
   
     "Your Committee on Canals, to which was referred House Bill No.246,
     entitled an act for the introduction of a mathematical truth, etc.,
     has had the same under consideration and begs leave to report the
     same back to the House with the recommendation that said bill be
     referred to the Committee on Education."
     
   
   
   The next day, the following article appeared in the Indianapolis
   Sentinel:
   
     "To SQUARE THE CIRCLE
     
     "Claims Made That This Old Problem Has Been Solved. "The bill
     telling how to square a circle, introduced in the House by
     Mr.Record, is not intended to be a hoax. Mr. Record knows nothing of
     the bill with the exception that he introduced it by request of
     Dr.Edwin Goodwin of Posey County, who is the author of the
     demonstration. The latter and State Superintendent of Public
     Instruction Geeting believe that it is the long-sought solution of
     the problem, and they are seeking to have it adopted by the
     legislature. Dr. Goodwin, the author, is a mathematician of note. He
     has it copyrighted and his proposition is that if the legislature
     will indorse the solution, he will allow the state to use the
     demonstration in its textbooks free of charge. The author is
     lobbying for the bill."
     
     On "February 2, 1897, ...Representative S.E. Nicholson, of Howard
     County, chairman of the Committee on Education, reported to the
     House.
     
     "Your Committee on Education, to which was referred House Bill
     No.246, entitled a a bill for an act entitled an act introducing a
     new mathematical truth, has had same under consideration, and begs
     leave to report the same back to the House with the recommendation
     that said bill do pass.
     
     "The report was cincurred in, and on February 8, 1897, it was
     brought up for the second reading, following which it was considered
     engrossed. Then 'Mr. Nicholson moved that the consitutional rule
     requiring bills to be read on three days be suspended, that the bill
     may be read a third time now.' The constitutional rule was suspended
     by a vote of 72 to 0 and the bill was then read a third time. It was
     passed by a vote of 67 to 0, and the Clerk of the House was directed
     to inform the Senate of the passage of the bill."
     
   
   
   The newspapers reported the suspension of the consitutional rules and
   the unanimous passage of the bill matter-of-factly, except for one
   line in the Indianapolis Journal to the effect that "this is the
   strangest bill that has ever passed an Indiana Assembly."
   
   The bill was referred to the Senate on Feb.10,1897 and was read for
   the first time on Feb.11 and referred to the Committee on Temperance.
   "On Feb.12 Senator Harry S. New, of Marion County, Chairman of the
   Committee on Temperance, made the following report to the Senate:
   
     "Your committee on Temperance, to which was referred House Bill
     No.246, introduced by Mr.Record, has had the same under
     consideration and begs leave to report the same back to the Senate
     with the recommendation that said bill do pass."
     
   
   
   The Senate Journal mentions only that the bill was read a second time
   on Feb.12, 1897, that there was an unsuccessful attempt to amend the
   bill by strike out the enacting clause, and finally it was postponed
   indefinitely. That the bill was killed appears to be a matter of dumb
   luck rather than the superior education or wisdom of the Senate. It is
   true that the bill was widely ridiculed in Indiana and other states,
   but what actually brought about the defeat of the bill is recorded by
   Prof. C.A. Waldo in an article he wrote for the Proceedings of the
   Indiana Academy of Science in 1916. The reason he knows is that he
   happened to be at the State Capitol lobbying for the appropriation of
   the Indiana Academy of Science, on the day the Housed passed House
   Bill 246. When he walked in the found the debate on House Bill 246
   already in progress. In his article, he writes (according to
   Edington):
   
     "An ex-teacher from the eastern part of the state was saying: 'The
     case is perfectly simple. If we pass this bill which establishes a
     new and correct value for pi , the author offers to our state
     without cost the use of his discovery and its free publication in
     our school text books, while everyone else must pay him a royalty.'
     "
     
   
   
   The roll was then called and the bill passed its third and final
   reading in the lower house. A member then showed the writer [i.e.
   Waldo] a copy of the bill just passed and asked him if he would like
   an introduction to the learned doctor, its author. He declined the
   courtesy with thanks remarking that he was acquainted with as many
   crazy people as he cared to know.
   
   "That evening the senators were properly coached and shortly
   thereafter as it came to its final reading in the upper house they
   threw out with much merriment the epoch making discovery of the Wise
   Man from the Pocket."
   
   The bill implies four different values for pi and one for sqrt(2) , as
   follows: pi' = 16/sqrt(3) , 2 sqrt(5 pi/6) , 16 sqrt(2)/7 , 16/5
   (~9.24 , ~3.236 , ~3.232 , 3.2 respectively.) sqrt(2)' = 10/7.
   
     It has been found that a circular area is to the square on a line
     equal to the quadrant of the circumference, as the area of an
     equilateral rectangle is to the square on one side.
     
   pi' = ((pi'/2))^2 = sqrt(3)/4 : 1 i.e. pi' = 16/sqrt(3) ~= 9.24 .
   
     The diameter employed as the linear unit according to the present
     rule in computing the circle's area is entirely wrong, as it
     represents the circles area one and one-fifths times the area of a
     square whose perimeter is equal to the circumference of the circle.
     This is because one-fifth of the diameter fils to be represented
     four times in the circle's circumference.
     
   
   
   Bit tricky to decipher, but it seems to say ((2 pi'/4))^26/5 = pi i.e.
   pi' = 2 sqrt(5 pi/6) ~= 3.236
   
     Furthermore, it has revealed the ratio of the chord and arc of
     ninety degrees, which is as seven to eight,
     
   
   
   sqrt(2) : pi/2 = 7 : 8 i.e. pi = 16 sqrt(2)/7 ~= 3.232
   
     and also the ratio of the diagonal and one side of a square which is
     as ten to seven
     
   
   
   i.e. sqrt(2) = 10/7 ~= 1.429
   
     that the ratio of the diameter and circumference is as five-fourths
     to four
     
   
   
   i.e. pi = 16/5 = 3.2
   
   
     _________________________________________________________________
   
   
   
    alopez-o@barrow.uwaterloo.ca
    Sun Nov 20 20:45:48 EST 1994


Archive-Name: sci-math-faq/pointers
Last-modified: December 8, 1994
Version: 6.1


The Sci.Math FAQ may be consulted at, which as usual contains the 
latest additions and modifications, which eventually make it to
the posted version.


URL: http://daisy.uwaterloo.ca/~alopez-o/math-faq/math-faq.html


This brings up an introductory page. Users with character only
browsers should select "the Text Table of Contents"

or go directly to it via

URL: http//daisy.uwaterloo.ca/~alopez-o/math-faq/mathtext/math-faq.html


All other users (i.e. Mosaic/Air Mosaic/Netscape/etc) just follow the links
at the top of the page.


Text equations for the lynx version were generated automagically from
the LaTeX source by LaTexT coming soon to a latex2html distibution
near you.






    alopez-o@barrow.uwaterloo.ca
    Sun Nov 20 20:45:48 EST 1994



 


Archive-Name: sci-math-faq/proyectiveplane
Last-modified: December 8, 1994
Version: 6.1

   
			PROJECTIVE PLANE OF ORDER 10
				    

   More precisely:
   
   Is it possible to define 111 sets (lines) of 11 points each such that:
   
   
   For any pair of points there is precisely one line containing them
   both and for any pair of lines there is only one point common to them
   both?
   
   Analogous questions with n^2 + n + 1 and n + 1 instead of 111 and 11
   have been positively answered only in case n is a prime power. For n =
   6 it is not possible, more generally if n is congruent to 1 or 2 mod 4
   and can not be written as a sum of two squares, then an FPP of order n
   does not exist. The n = 10 case has been settled as not possible
   either by Clement Lam. As the ``proof" took several years of computer
   search (the equivalent of 2000 hours on a Cray-1) it can be called the
   most time-intensive computer assisted single proof. The final steps
   were ready in January 1989.
   
   
   
   References
   
   R. H. Bruck and H. J. Ryser. The nonexistence of certain finite
   projective planes. Canadian Journal of Mathematics, vol. 1 (1949), pp
   88-93.
   
   
   
   C. Lam. American Mathematical Monthly, 98 (1991), 305-318.
   
   
     _________________________________________________________________
   
   
   
    alopez-o@barrow.uwaterloo.ca
    Sun Nov 20 20:45:48 EST 1994


Archive-Name: sci-math-faq/primes
Last-modified: December 8, 1994
Version: 6.1


				PRIME NUMBERS
				       
 
  Contents:

     _________________________________________________________________
   
     * Largest known Mersenne prime
     * Largest known prime
     * Largest known twin primes
     * Largest Fermat number with known factorization
     * Algorithms to factor integer numbers
     * List of record numbers
     * What is the current status on Mersenne primes?
     * Formulae to compute prime numbers
       
   
     _________________________________________________________________
   
  
 
   
Largest known Mersenne prime

   
   
   2^(859433) - 1 is prime. It was discovered in 1994.
   
   
     _________________________________________________________________
   
   
   
Largest known prime

   
   
   The largest known prime is the Mersenne prime described above. The
   largest known non-Mersenne prime, is 391581*2^(216193) - 1 .
   Throughout history, the largest known prime has almost always been a
   Mersenne prime; the period between Brown et al's discovery in Aug 1989
   and Slowinski & Gage's in March 1992 is one of the few exceptions.
   
   
   
   References
   
   Brown, Noll, Parady, Smith, Smith, and Zarantonello. Letter to the
   editor. American Mathematical Monthly, vol. 97, 1990, p. 214.
   
   
   
     _________________________________________________________________
   
Largest known twin primes

   
   
   The largest known twin primes are 1691232 * 1001 * 10^(4020) +- 1 ,
   which is a number with 4030 digits, found by H. Dubner. The second
   largest known twin primes are 4650828 * 1001 * 10^(3429) +- 1 . They
   were found by H. Dubner as well.
   
   
   
   References
   
   B. K. Parady and J. F. Smith and S. E. Zarantonello, Smith, Noll and
   Brown. Largest known twin primes. Mathematics of Computation, vol.55,
   1990, pp. 381-382.
   
   
     _________________________________________________________________
   
Largest Fermat number with known factorization

   
   
   F_(11) = (2^(2^(11))) + 1 which was factored by Brent &Morain in 1988.
   F_9 = (2^(2^9)) + 1 = 2^(512) + 1 was factored by A.K. Lenstra, H.W.
   Lenstra Jr., M.S. Manasse &J.M. Pollard in 1990. The factorization for
   F_(10) is not known.
   
  

     _________________________________________________________________
 
Algorithms to factor integer numbers

   
   
   There are several know algorithms that have subexponential estimated
   running time, to mention just a few:
   
     * Continued fraction algorithm.
     * Class group method.
     * Quadratic sieve algorithm.
     * Class Group method.
     * Elliptic curve algorithm.
     * Number fi eld sieve.
     * Dixon's random squares algorithm.
     * Valle's two-thirds algorithm.
     * Seysen's class group algorithm.
       
   
   
   
   
   References
   
   A.K. Lenstra, H.W. Lenstra Jr. Algorithms in Number Theory. J. van
   Leeuwen (ed.), Handbook of Theoretical Computer Science, Volume A:
   Algorithms and Complexity Elsevier, pp. 673-715, 1990.
   
   
   
   
     _________________________________________________________________
   
   

List of record numbers

   
   
   Chris Caldwell maintains a list called The Largest Known Primes.
   Finger primes@math.utm.edu for a few record primes and ways to get the
   lists by e-mail, the list is also available by e-mail. He can e-mail
   you the lists, but prefers you use the other methods. different forms
   of this list. Currently the list is available by anonymous ftp to
   math.utm.edu (directory /pub/math/primes) and by gopher to
   unix1.utm.edu (directory 1/user/Public_FTP/pub/math/primes). If
   nothing else works, you may send e-mail to Chris for a copy of the
   lists. (caldwell@utm.edu or caldwell@utmartn.bitnet).
   
   
     _________________________________________________________________
   
   
   
What is the current status on Mersenne primes?

   
   
   Mersenne primes are primes of the form 2^p - 1 . For 2^p - 1 to be
   prime we must have that p is prime. The following Mersenne primes are
   known.

  Number        p                        Year   Discoverer

     1-4     2,3,5,7                 pre-1500
     5          13                       1461   Anonymous
     6-7        17,19                    1588   Cataldi
     8          31                       1750   Euler
     9          61                       1883   I.M. Pervushin
    10          89                       1911   Powers
    11          107                      1914   Powers
    12          127                      1876   Lucas
    13-14       521,607                  1952   Robinson
    15-17       1279,2203,2281           1952   R. M. Robinson
    18          3217                     1957   Riesel
    19-20       4253,4423                1961   Hurwitz & Selfridge
    21-23       9689,9941,11213          1963   Gillies
    24          19937                    1971   Tuckerman
    25          21701                    1978   Noll & Nickel
    26          23209                    1979   Noll
    27          44497                    1979   Slowinski & Nelson
    28          86243                    1982   Slowinski
    29          110503                   1988   Colquitt & Welsh
    30          132049                   1983   Slowinski
    31          216091                   1985   Slowinski
    32?         756839                   1992   Slowinski & Gage
    33?         859433                   1994   Slowinski

   
   
   
   
   The way to determine if 2^p - 1 is prime is to use the Lucas-Lehmer
   test:
   

      Lucas_Lehmer_Test(p):
	 u := 4
	 for i from 3 to p do
	    u := u^2-2 mod 2^p-1
	 od
	 if u == 0 then
	    2^p-1 is prime
	 else
	    2^p-1 is composite
	 fi

   
   
   The following ranges have been checked completely: 2 - 355K,
   360K-386K, and 430K - 520K.
   
   
   
   References
   
   An introduction to the theory of numbers. G.H. Hardy, E.M. Wright.
   Fifth edition, 1979, Oxford.
   
   
   
   
     _________________________________________________________________
   
   
   
Formulae to compute prime numbers

   
   
   Is there a polynomial which gives all the prime numbers?
   
   No, there is not. This is a simple exercise to prove.
   
   Is there a non-constant polynomial that only takes on prime values?
   
   It has been proved that no such polynomial exists. The proof is simple
   enough that an high school student could probably discover it. See,
   for example, Ribenboim's book The Book of Prime Number Records.
   
   Note, however, by the work of Jones, Sato, Wada, and Wiens, there is a
   polynomial in 26 variables such that the set of primes coincides with
   the set of positive values taken by this polynomial. See Ribenboim,
   pp. 147-150.
   
   But most people would object to the term ``formula" restricted to mean
   polynomial. Can we not use summation signs, factorial, and the floor
   function in our ``formula"? If so, then indeed, there are formulas for
   the prime numbers. Some of them are listed below.
   
   If we can't, then exactly what operations do you allow and why?
   
   Indeed, as I have previously argued, a reasonable interpretation of
   the word ``formula" is simply ``Turing machine that halts on all
   inputs". Under this interpretation, there certainly are halting Turing
   machines which compute the n -th prime number. However, nobody knows
   how to compute the n -th prime in time polynomial in log n . That's
   still an open question.
   
   Herb Wilf has addressed the question, ``What is a formula?" in his
   delightful article, ``What is an answer?" which appeared in the
   American Mathematical Monthly, 89 (1982), 289-292. He draws the
   distinction between ``formula" and ``good formula". Anyone who claims
   ``there is no formula for the prime numbers" should read this article.
   
   
   Here are just a few articles that discuss ``formulas" for primes.
   Almost all of these do not require computation of the primes "ahead of
   time". Most of them rely on standard mathematical functions such as
   summation, factorial, greatest integer function, etc.
   
   
   
   References
   
   C. Isenkrahe. Math. Annalen 53 (1900), 42-44.
   
   
   
   W. H. Mills. Bulletin of the American Mathematical Society 53 (1947),
   604.
   
   
   
   L. Moser. Mathematics Magazine 23 (1950), 163-164.
   
   
   
   E. M. Wright. American Mathematical Monthly 58 (1951), 616-618.
   (Correction, 59 (1952), 99.)
   
   
   
   E. M. Wright. Journal of the London Mathematical Society 29 (1954),
   63-71.
   
   
   
   B. R. Srinivasan. Journal of the Indian Mathematical Society 25
   (1961), 33-39.
   
   
   
   C. P. Willans. Mathematics Gazette 48 (1964), 413-415.
   
   
   
   V. C. Harris. Nordisk Mat. Tidskr. 17 (1969), 82.
   
   
   
   U. Dudley. American Mathematical Monthly 76 (1969), 23-28.
   
   
   
   C. Vanden Eynden. American Mathematical Monthly 79 (1972), 625.
   
   
   
   S. W. Golomb. American Mathematical Monthly 81 (1974), 752-754.
   
   
   
   
   
   Algorithmic Number Theory J.O. Shallit, E. Bach. (to be published, MIT
   Press).
   
   
   
   
   
   
     _________________________________________________________________
   
   
Archive-Name: sci-math-faq/Quaternions
Last-modified: December 8, 1994
Version: 6.1



		  THEORY OF QUATERNIONIC ANALYTIC FUNCTIONS
				       
   
   
   Four-dimensional analog to the theory of complex analytic functions.
   It was developed in the 1930s by the mathematician Fueter. It is based
   on a generalization of the Cauchy-Riemann equations, since the
   possible alternatives of power series expansions or quaternion
   differentiability do not produce useful theories. A number of useful
   integral theorems follow from the theory. Sudbery provides an
   excellent review. Deavours covers some of the same material less
   thoroughly. Brackx discusses a further generalization to arbitrary
   Clifford algebras.
   
   
   
   References
   
   Anthony Sudbery. Quaternionic Analysis. Proc. Camb. Phil. Soc., vol.
   85, pp 199-225, 1979.
   
   
   
   Cipher A. Deavours. The Quaternion Calculus. Am. Math. Monthly, vol.
   80, pp 995-1008, 1973.
   
   
   
   Clifford analysis. F. Brackx and R. Delanghe and F. Sommen. Pitman,
   1983.
   
   
   
   
   
   
     _________________________________________________________________
   
   
   
    alopez-o@barrow.uwaterloo.ca
    Sun Nov 20 20:45:48 EST 1994

Archive-Name: sci-math-faq/surfaceSphere
Last-modified: December 8, 1994
Version: 6.1

   
Formula for the Surface Area of a sphere in Euclidean N -Space

   
   This is equivalent to the volume of the N -1 solid which comprises the
   boundary of an r^N (pi^(N/2))/((N/2)!) -Sphere.
   
   The volume of a ball is the easiest formula to remember: It's x! =
   Gamma (x + 1) . The only hard part is taking the factorial of a
   half-integer. The real definition is that (1/2 + n)! = sqrt(pi) ((2n +
   2)!)/((n + 1)!4^(n + 1)) , but if you want a formula, it's:
   
   N (pi^(N/2))/((N/2)!r^(N - 1)) To get the surface area, you just
   differentiate to get e^(-x^2) .
   
   There is a clever way to obtain this formula using Gaussian integrals.
   First, we note that the integral over the line of sqrt(pi) is N .
   Therefore the integral over e^(-x_1^2 - x_2^2 - ... - x_N^2) -space of
   sqrt(pi)^n is Vr^(N - 1)e^(-r^2) . Now we change to spherical
   coordinates. We get the integral from 0 to infinity of V , where n is
   the surface volume of a sphere. Integrate by parts repeatedly to get
   the desired formula.
   
   It is possible to derive the volume of the sphere from ``first
   principles''.
   
   
     _________________________________________________________________
   
   
   
    alopez-o@barrow.uwaterloo.ca
    Sun Nov 20 20:45:48 EST 1994


Archive-Name: sci-math-faq/symboliccomp    
Last-modified: December 8, 1994
Version: 6.1

   
			SYMBOLIC COMPUTATION PACKAGES
				       
   
   
   This is not a comprehensive list. There are other Computer Algebra
   packages available that may better suit your needs. There is also a
   FAQ list in the group sci.math.symbolic. It includes a much larger
   list of vendors and developers. (The FAQ list can be obtained from
   math.berkeley.edu via anonymous ftp).
   

A: Maple
	Purpose: Symbolic and numeric computation, mathematical
	programming, and mathematical visualization.
	Contact: Waterloo Maple Software,
	450 Phillip Street
	Waterloo, Ontario
	N2L 5J2
	Phone (519)747-2373
	FAX   (519)747-5284
	email:  info@maplesoft.on.ca
A: DOE-Macsyma
	Purpose: Symbolic and mathematical manipulations.
	Contact: National Energy Software Center
	Argonne National Laboratory 9700 South Cass Avenue
	Argonne, Illinois 60439
	Phone: (708) 972-7250
A: Pari
	Purpose: Number-theoretic computations and simple numerical
	analysis.
	Available for most 32-bit machines, including 386+387 and 486.
	This is a copyrighted but free package, available by ftp from
	math.ucla.edu (128.97.4.254) and ftp.inria.fr (128.93.1.26).
	Contact: questions about pari can be sent to pari@ceremab.u-bordeaux.fr
	and for the Macintosh versions to bernardi@mathp7.jussieu.fr
A: Mathematica
	Purpose: Mathematical computation and visualization,
	symbolic programming.
	Contact: Wolfram Research, Inc.
	100 Trade Center Drive Champaign,
	IL 61820-7237
	Phone: 1-800-441-MATH
	info@wri.com
A: Macsyma
	Purpose: Symbolic numerical and graphical mathematics.
	Contact: Macsyma Inc.
	20 Academy Street
	Arlington, MA 02174
	tel: 617-646-4550
	fax: 617-646-3161
	email: info-macsyma@macsyma.com
A: Matlab
	Purpose: `matrix laboratory' for tasks involving
	matrices, graphics and general numerical computation.
	Contact: The MathWorks, Inc.
	21 Prime Park Way
	Natick, MA 01760
	508-653-1415
	info@mathworks.com
A: Cayley
	Purpose: Computation in algebraic and combinatorial structures
	such as groups, rings, fields, modules and graphs.
	Available for: SUN 3, SUN 4, IBM running AIX or VM, DEC VMS, others
	Contact: Computational Algebra Group
	University of Sydney
	NSW 2006
	Australia
	Phone:  (61) (02) 692 3338
	Fax: (61) (02) 692 4534
	cayley@maths.su.oz.au

   
   
   
   
   
     _________________________________________________________________
   
   
   
    alopez-o@barrow.uwaterloo.ca
    Sun Nov 20 20:45:48 EST 1994


Archive-Name: sci-math-faq/unsolvedproblems
Last-modified: December 8, 1994
Version: 6.1

			      UNSOLVED PROBLEMS
				       
   
     _________________________________________________________________
   
     * Does there exist a number that is perfect and odd?
     * Collatz Problem
       
   
     _________________________________________________________________
   
   
   
Does there exist a number that is perfect and odd?

   
   
   A given number is perfect if it is equal to the sum of all its proper
   divisors. This question was first posed by Euclid in ancient Greece.
   This question is still open. Euler proved that if N is an odd perfect
   number, then in the prime power decomposition of N , exactly one
   exponent is congruent to 1 mod 4 and all the other exponents are even.
   Furthermore, the prime occurring to an odd power must itself be
   congruent to 1 mod 4. A sketch of the proof appears in Exercise 87,
   page 203 of Underwood Dudley's Elementary Number Theory, 2nd ed. It
   has been shown that there are no odd perfect numbers < 10^(300) .
   
   
     _________________________________________________________________
   
   
   
Collatz Problem

   
   
   Take any natural number m > 0 .
   n : = m;
   repeat
   if ( n is odd) then n : = 3*n + 1 ; else n : = n/2 ;
   until ( n = = 1 )
   
   
   Conjecture. For all positive integers m, the program above terminates.
   
   
   
   
   The conjecture has been verified up to 7 * 10^(11) .
   
   
   
   References
   
   Unsolved Problems in Number Theory. Richard K Guy. Springer, Problem
   E16.
   
   
   
   
   
   
     _________________________________________________________________
   
   
   
    alopez-o@barrow.uwaterloo.ca
    Sun Nov 20 20:45:48 EST 1994


Archive-Name: sci-math-faq/FLT/Wiles
Last-modified: December 8, 1994
Version: 6.1

   
Wiles' line of attack

   
   
   Here is an outline of the first proposed proof.
   
	  From Ken Ribet:
	  
	  Here is a brief summary of what Wiles said in his three
	  lectures.
	  
	  The method of Wiles borrows results and techniques from lots
	  and lots of people. To mention a few: Mazur, Hida, Flach,
	  Kolyvagin, yours truly, Wiles himself (older papers by Wiles),
	  Rubin... The way he does it is roughly as follows. Start with a
	  mod p representation of the Galois group of Q which is known to
	  be modular. You want to prove that all its lifts with a certain
	  property are modular. This means that the canonical map from
	  Mazur's universal deformation ring to its maximal Hecke algebra
	  quotient is an isomorphism. To prove a map like this is an
	  isomorphism, you can give some sufficient conditions based on
	  commutative algebra. Most notably, you have to bound the order
	  of a cohomology group which looks like a Selmer group for Sym^2
	  of the representation attached to a modular form. The
	  techniques for doing this come from Flach; you also have to use
	  Euler systems a la Kolyvagin, except in some new geometric
	  guise.
	  
	  If you take an elliptic curve over Q , you can look at the
	  representation of Gal on the 3-division points of the curve. If
	  you're lucky, this will be known to be modular, because of
	  results of Jerry Tunnell (on base change). Thus, if you're
	  lucky, the problem I described above can be solved (there are
	  most definitely some hypotheses to check), and then the curve
	  is modular. Basically, being lucky means that the image of the
	  representation of Galois on 3-division points is GL(2,Z/3Z) .
	  
	  Suppose that you are unlucky, i.e., that your curve E has a
	  rational subgroup of order 3. Basically by inspection, you can
	  prove that if it has a rational subgroup of order 5 as well,
	  then it can't be semistable. (You look at the four non-cuspidal
	  rational points of X_0(15) .) So you can assume that E[5] is
	  ``nice''. Then the idea is to find an E' with the same
	  5-division structure, for which E'[3] is modular. (Then E' is
	  modular, so E'[5] = E[5] is modular.) You consider the modular
	  curve X which parameterizes elliptic curves whose 5-division
	  points look like E[5] . This is a twist of X(5) . It's
	  therefore of genus 0, and it has a rational point (namely, E ),
	  so it's a projective line. Over that you look at the
	  irreducible covering which corresponds to some desired
	  3-division structure. You use Hilbert irreducibility and the
	  Cebotarev density theorem (in some way that hasn't yet sunk in)
	  to produce a non-cuspidal rational point of X over which the
	  covering remains irreducible. You take E' to be the curve
	  corresponding to this chosen rational point of X .
	  
   
	  From the previous version of the FAQ:
	  
	  (b) conjectures arising from the study of elliptic curves and
	  modular forms. - The Taniyama-Weil-Shmimura conjecture.
	  
	  There is a very important and well known conjecture known as
	  the Taniyama-Weil-Shimura conjecture that concerns elliptic
	  curves. This conjecture has been shown by the work of Frey,
	  Serre, Ribet, et. al. to imply FLT uniformly, not just
	  asymptotically as with the ABC conjecture.
	  
	  The conjecture basically states that all elliptic curves can be
	  parameterized in terms of modular forms.
	  
	  There is new work on the arithmetic of elliptic curves. Sha,
	  the Tate-Shafarevich group on elliptic curves of rank 0 or 1.
	  By the way an interesting aspect of this work is that there is
	  a close connection between Sha, and some of the classical work
	  on FLT. For example, there is a classical proof that uses
	  infinite descent to prove FLT for n = 4 . It can be shown that
	  there is an elliptic curve associated with FLT and that for n =
	  4 , Sha is trivial. It can also be shown that in the cases
	  where Sha is non-trivial, that infinite-descent arguments do
	  not work; that in some sense ``Sha blocks the descent''.
	  Somewhat more technically, Sha is an obstruction to the
	  local-global principle [e.g. the Hasse-Minkowski theorem].
	  
   
	  From Karl Rubin:
	  
	  
	  
	  Theorem 2 If E is a semistable elliptic curve defined over Q,
	  then E is modular.
	  
	  
	  
	  It has been known for some time, by work of Frey and Ribet,
	  that Fermat follows from this. If u^q + v^q + w^q = 0 , then
	  Frey had the idea of looking at the (semistable) elliptic curve
	  y^2 = x(x - a^q)(x + b^q) . If this elliptic curve comes from a
	  modular form, then the work of Ribet on Serre's conjecture
	  shows that there would have to exist a modular form of weight 2
	  on Gamma_0(2) . But there are no such forms.
	  
	  To prove the Theorem, start with an elliptic curve E , a prime
	  p and let rho_p : Gal(\bar(Q)/Q) --> GL_2(Z/pZ) be the
	  representation giving the action of Galois on the p -torsion
	  E[p] . We wish to show that a certain lift of this
	  representation to GL_2(Z_p) (namely, the p -adic representation
	  on the Tate module T_p(E) ) is attached to a modular form. We
	  will do this by using Mazur's theory of deformations, to show
	  that every lifting which ``looks modular'' in a certain precise
	  sense is attached to a modular form.
	  
	  Fix certain ``lifting data'', such as the allowed ramification,
	  specified local behavior at p , etc. for the lift. This defines
	  a lifting problem, and Mazur proves that there is a universal
	  lift, i.e. a local ring R and a representation into GL_2(R)
	  such that every lift of the appropriate type factors through
	  this one.
	  
	  Now suppose that rho_p is modular, i.e. there is some lift of
	  rho_p which is attached to a modular form. Then there is also a
	  hecke ring T , which is the maximal quotient of R with the
	  property that all modular lifts factor through T . It is a
	  conjecture of Mazur that R = T , and it would follow from this
	  that every lift of rho_p which ``looks modular'' (in particular
	  the one we are interested in) is attached to a modular form.
	  
	  Thus we need to know 2 things:
	(a)
	    rho_p is modular
	(b)
	    R = T .
   
	  
	  It was proved by Tunnell that rho_3 is modular for every
	  elliptic curve. This is because PGL_2(Z/3Z) = S_4 . So (a) will
	  be satisfied if we take p = 3 . This is crucial.
	  
	  Wiles uses (a) to prove (b) under some restrictions on rho_p .
	  Using (a) and some commutative algebra (using the fact that T
	  is Gorenstein, basically due to Mazur) Wiles reduces the
	  statement T = R to checking an inequality between the sizes of
	  2 groups. One of these is related to the Selmer group of the
	  symmetric square of the given modular lifting of rho_p , and
	  the other is related (by work of Hida) to an L -value. The
	  required inequality, which everyone presumes is an instance of
	  the Bloch-Kato conjecture, is what Wiles needs to verify.
	  
	  He does this using a Kolyvagin-type Euler system argument. This
	  is the most technically difficult part of the proof, and is
	  responsible for most of the length of the manuscript. He uses
	  modular units to construct what he calls a geometric Euler
	  system of cohomology classes. The inspiration for his
	  construction comes from work of Flach, who came up with what is
	  essentially the bottom level of this Euler system. But Wiles
	  needed to go much farther than Flach did. In the end, under
	  certain hypotheses on rho_p he gets a workable Euler system
	  and proves the desired inequality. Among other things, it is
	  necessary that rho_p is irreducible.
	  
	  Suppose now that E is semistable.
	  
	  Case 1. rho_3 is irreducible.
	  Take p = 3. By Tunnell's theorem (a) above is true. Under these
	  hypotheses the argument above works for rho_3 , so we conclude
	  that E is modular.
	  
	  Case 2. rho_3 is reducible. Take p = 5 . In this case rho_5
	  must be irreducible, or else E would correspond to a rational
	  point on X_0(15) . But X_0(15) has only 4 noncuspidal rational
	  points, and these correspond to non-semistable curves. If we
	  knew that rho_5 were modular, then the computation above would
	  apply and E would be modular.
	  
	  We will find a new semistable elliptic curve E' such that
	  rho_(E,5) = rho_(E',5) and rho_(E',3) is irreducible. Then by
	  Case I, E' is modular. Therefore rho_(E,5) = rho_(E',5) does
	  have a modular lifting and we will be done.
	  
	  We need to construct such an E' . Let X denote the modular
	  curve whose points correspond to pairs (A, C) where A is an
	  elliptic curve and C is a subgroup of A isomorphic to the group
	  scheme E[5] . (All such curves will have mod-5 representation
	  equal to rho_E .) This X is genus 0, and has one rational point
	  corresponding to E , so it has infinitely many. Now Wiles uses
	  a Hilbert Irreducibility argument to show that not all rational
	  points can be images of rational points on modular curves
	  covering X , corresponding to degenerate level 3 structure
	  (i.e. im(rho_3) != GL_2(Z/3) ). In other words, an E' of the
	  type we need exists. (To make sure E' is semistable, choose it
	  5-adically close to E . Then it is semistable at 5, and at
	  other primes because rho_(E',5) = rho_(E,5) .)
	  
   
   
   
     _________________________________________________________________
   
   
   
    alopez-o@barrow.uwaterloo.ca
    Sun Nov 20 20:45:48 EST 1994